You are on page 1of 180

Instructors Manual/Test Bank

to accompany
Wassermans

The Basics of American Politics


Eleventh Edition

Larry Elowitz Georgia College and State University

New York Boston San Francisco London Toronto Sydney Tokyo Singapore Madrid Mexico City Munich Paris Cape Town Hong Kong Montreal

Instructors Manual/Test Bank to accompany Wasserman, Basics of American Politics, Eleventh Edition Copyright 2004 Pearson Education All rights reserved. Printed in the United States of America. Instructors may reproduce portions of this book for classroom use only. All other reproductions are strictly prohibited without prior permission of the publisher, except in the case of brief quotations embodied in critical articles and reviews. ISBN: 0-321-18837-3 1 2 3 4 5 6 7 8 9 10-DPC 06 05 04 03

TABLE OF CONTENTS
Chapter 1 Chapter 2 Chapter 3 Chapter 4 Chapter 5 Chapter 6 Chapter 7 Chapter 8 Chapter 9 What is Politics? The Constitution: Rules of the Game The Executive Branch: The President and Bureaucracy The Legislative Branch: Congress The Judicial Branch: The Supreme Court and the Federal Court System Civil Rights and Liberties: Protecting the Players Voters and Political Parties Interest Groups and the Media Who Wins, Who Loses: Pluralism Versus Elitism 1 16 36 58 82 101 123 145 166

CHAPTER ONE What is Politics?

CHAPTER OVERVIEW This initial chapter introduces and defines politics and applies it to America's government. In Lasswell's famous definition, politics is "the process of who gets what, when, and how." In other words, the text's definition of politics "centers on actions among a number of people involving influence (note the opening example of the student-professor classroom interaction). The concepts of power (the ability to influence another's behavior), elites (those who get more than others of the values society has available), legitimacy (the acceptance of something as right), and authority (a legitimate, efficient form of power) are discussed. The means by which democratic politics chooses conciliation rather than violence to maintain order and adapt to change, as well as the fact that widespread participation though freely-elected representatives is the cornerstone of every democracy, is also explored. The responsibilities of government are investigated; in particular, the need for government as a means of controlling conflict (disputes over distributions of a society's valued things), making rules determining who gets the valued things of society, and regulating the use of "legitimate force." A condition of anarchy, or a society without government, is shown to be unrealistic. Political science, a discipline that studies the power-authority interaction among people, is then distinguished from the other social sciences, such as economics or history, by looking at Microsoft as an arena for these different approaches. Political scientists would look at how Bill Gates and his executives make decisions, the impact of the government's antitrust 1

suit against Microsoft, how the computer giant maintains its business preeminence, and how/why its leaders maintain power and authority. In short, to quote Lasswell once again, politics "is the study of influence and the influential." The author illustrates the life and death consequences of the political game with the example of the FAA's failure to regulate a door problem on the DC-10. Political considerations resulted in needed safety modifications for cargo doors being delayed by McDonnell Douglas, the DC-10's manufacturer. The result was a cargo door blowing off on a Turkish-owned DC-10 in1974, resulting in the deaths of 346 people on board. The author concludes this section by explaining the organization of subsequent chapters and the reasons why we should all give a damn about the "complex, ever changing, never ending, and serious" game of politics. In short, he says,"from the moment we wake up in the morning, we are affected by someone's political choices." LEARNING OBJECTIVES After reading chapter 1, the student should be able to: 1. Define "politics" and explain why it is so important in American society. Be sure to analyze the Lasswell definitions. Also, analyze the political significance of the "Guess Who's Coming to Dinner?" boxed insert, especially from the perspective of "equality." 2. Explain the significance of power, authority, and political elites. Also, explain why U.S. power was "limited" regarding Saddam Hussein's removal from power in 1991. 3. Summarize the reasons why government is necessary, as well as its two key functions. Also, how and why does government in the U.S. both protect the private distribution of most of society's values and simultaneously set limits on that private distribution? Explain how government "alone" regulates the use of legitimate force. 2

4. Understand how democratic politics differs from rival approaches and what is meant by representative democracy. Also, explain how politics in a democracy behaves like a marketplace. 5. Distinguish what political scientists study from what other disciplines-history, economics, or psychology--study. In addition, summarize how political scientists would study Microsoft. 6. Appreciate the serious and complex nature of the "political game" and understand how and why politics affects the individual's life in so many ways. 7. Review how politics was a "life and death" affair in the case of the FAA, McDonnell Douglas, and the DC-10 cargo door catastrophe. OUTLINE I. What is Politics? -- The First Day of Class A. Classroom example. A very familiar illustration of how people get and use influence. Representative of Lasswell's classic definition of politics as "the process of who gets what, when, and how." II. Politics and Power A. Power Defined. Power is the ability to influence another's behavior. A distinction is made between the capacity for power (e.g., a large military) and the ability to apply it (e.g., the American problem with Saddam Hussein in Iraq). Power is generally a means to other ends; a method to acquire values, such as wealth.

B. Elites--Groups that wield power. The elite is defined as that group which has most of the resources (both material and nonmaterial) that society values. The unequal distribution of wealth in America is used as an example and is supported with statistics. For example, the top 1 percent of the population owns 33 percent of the wealth. Elites may differ according to the value considered (wealth, respect, etc.), but generally possession of one value leads to control of other values. C. Authority: Legitimate Power. Authority is defined as legitimate power--it includes not only legal obligations, but also psychological and moral ones. Most people feel they should obey an authority; therefore, the need for force is minimized, making authority an efficient form of power. See Guess Whos Coming to Dinner? III. The Need for Government A. Why Government? Government is necessary to settle inevitable conflicts over the distribution of valued things. Government itself often becomes a potential threat, and its enormous power must be controlled. The American system of constitutional, representative, democratic government recognizes this need to limit government. Democracy chooses conciliation rather than violence, i.e., no group is viewed as having a monopoly on the truth. B. What is Government? Government does three things: 1. It makes the rules determining who gets the things valued by society. This may mean the theoretical noninterference of the Constitution in the private economy; or the actual use of government power to limit the private distribution of wealth, by such devices as income taxes and welfare; or direct government intervention in disputes among citizens. 2. Representative democracy allows people to effectively participate in governing through elected representatives. 4

3.

An essential part of democracy is tolerating differing opinions and interests and seeking compromises among competing demands. Government alone regulates the use of legitimate force, by employing, allowing, or preventing it. Government does not monopolize the use of legitimate force, but it alone sets limits to its use.

IV. The Study of Politics A. Political Science as a Discipline. Politics is grouped with the other social sciences as the study of interactions among people, but it is distinguished by its primary interest in the concepts of power and authority. A case study of the "society" of Microsoft is used to illustrate these differences. B. Political Science and Microsoft. An economist, psychologist, and historian would all ask different questions about the computer software giant, Microsoft (questions could overlap). The political scientist would focus on the central question-who is getting what, when, and how? C. Why Give a Damn About Politics? Apathy is also a political position because it affects who gets what, and when and how. A tragic air accident with personal consequences, attributed to the negligence of a government agency, illustrates the effects of political decisions. Also, see Who Needs Government? V. What is This Book About? A brief outline of the chapters is given and the analogy of politics to a game is discussed. Politics is a game in form, but not in intent. It is deadly serious, played for high and varied stakes, and is highly complex, involving several overlapping games, in which various groups contend for power to gain their values and protect their interests. 5

KEY CHAPTER TERMS AND IDENTIFICATIONS Politics Harold Lasswell's definitions of politics "who gets what, when, and how" power Saddam Hussein/Iraq in 1991 authority as "legitimate power" a "capacity for power" political elites/distribution of wealth anarchy political conflict democracy representative democracy Bernard Crick's definition of democratic politics free elections Tutsis of Rwanda Supreme Court and the 2000 election government values "regulation of legitimate force" social sciences--history, economics, sociology, psychology political scientists/Microsoft and Bill Gates "the study of influence and the influential" Senator Ernest Hollings and "Who Needs Government"? The FAA, McDonnell Douglas, and the DC-10 the "game" of politics "arenas" of politics politics as a "spectator sport" TEACHING SUGGESTIONS 1. Discuss how a political scientist might outline a study of the operations of a large university. What decisions would he/she examine? What would be some of the objectives of the study? How 6

would it differ from other disciplines approaches? 2. Ask the class to speculate on whether "pure democracy" would work in America. Imagine if Americans had a device attached to their TV sets and could key in a "Yes" or "No" response to polls on an issue posed by local or national leaders. Would this be a good idea or not? Discuss. 3. Ask students whether politics has been a "spectator sport" for most of their lives. Assuming it has been, discuss why this has been the case and what factors may change this political attitude in the future. 4. Discuss whether the obvious inequality of income in America is dangerous to democracy. In what ways is this inequality an inevitable part of capitalism? Should this inequality be reduced through active intervention by the federal government--why or why not? SUGGESTED RESEARCH ASSIGNMENTS 1. Which elites in your community wield the greatest power? Looking at a recent local decision--zoning, education, or regulation--examine who contacted local decision-makers and what influenced the policy outcome. 2. It may be interesting to have students write a short paper explaining how they and/or their families have been affected by government agencies at the local, state, or federal levels. How did they react? Include whether these "government contacts" were positive or negative. 3. As a follow-up research project, have a team of students investigate the final outcome of the government's suit against Microsoft. What did this outcome reveal about the power of government versus the power of a giant corporation? 7

4. Ask students to interpret Senator Holling's story about the irate taxpayer in the "Who Needs Government?" boxed insert. Then, have those same students interview friends, family members, or relatives about whether taxes paid to government are used appropriately or not. Findings could be presented in the form of a paper or class panel report. MULTIPLE-CHOICE QUESTIONS 1. Harold Lasswell defines politics as a. relationships among people involving power and authority. b. a process of who gets what, when and how. c. the distribution of a societys wealth. d. all of the above Power fundamentally involves __________between or among people. a. friendships b. shared property c. animosities d. relationships The elites in a society a. are the people who get a disproportionately large part of the societys valued resources. b. comprise the majority of the population. c. are all college educated. d. occupy government positions. Why does the author assert that authority is "an efficient form of power"? a. authority is distributed equally among political elites b. authority is backed by legal norms c. authority, once accepted, is less costly than force d. none of the above is correct 8

2.

3.

4.

5.

Anarchy is best defined as a. unlimited democracy. b. elected representatives run the government. c. rule by an elite. d. lack of government. A government a. makes rules to determine who will get the valued things in society. b. is always democratic. c. is rarely coercive. d. protects the wealthy. The study of "influence and the influential" most clearly falls under the social science of a. history. b. economics. c. sociology. d. political science. Welfare programs are an example of a. unlimited government spending. b. the way government regulates the distribution of wealth. c. the system of checks and balances. d. all of the above A historian would study Microsoft primarily to determine a. who makes the important corporate decisions. b. the origins and development of the company. c. the cost-effectiveness of software production. d. the anti-trust demands of government against management.

6.

7.

8.

9.

10. A person who studies an organizations decision-making procedures is probably a. an economist. b. a political scientist. c. a psychologist. d. a historian. 11. In theory, the policy of the Constitution of the United States toward the distribution of wealth is a. to elevate state over federal control. b. to establish itself as the supreme and sole authority. c. to institute a central planning agency. d. noninterference. 12. Which of the following is an example of government intervention? a. providing a sound currency b. protection from domestic unrest c. safeguarding private property d. all of these 13. The opening dialogue of the book illustrates that a. students have no power. b. politics occurs in the classroom. c. students are apathetic about national politics. d. teachers seldom exercise their authority. 14. The top 1 percent of the American population possesses what percentage of the nation's wealth? a. 20 b. 27 c. 33 d. 39

10

15. The statement that the "United States is the most powerful nation in the world" actually means that America a. can force any nation to heed its wishes. b. has the will to dominate the globe but lacks resources. c. has a capacity for power that automatically translates into actual influence. d. none of the above. 16. The author defines power as the a. victory of the strong over the weak. b. ability to influence another's behavior. c. strength and size of a nation's armed forces. d. use of force and coercion to achieve one's goals. 17. Authority is a. the use of force over a weak opponent. b. legitimate power. c. coercive power. d. personified in the commander in chief.

18. When political conflict occurs, a. a new government may be formed as a result. b. government channels the conflict. c. democracies suppress those conflicts by force. d. both a and b are correct. 19. The tolerance of different opinions and interests is a. an essential part of democracy. b. needed in any corporation. c. essential to all ideologies. d. the reason some people have great power over others.

11

20. According to the author, the best word to describe the "political game" in America is a. simple. b. stable. c. serious. d. peripheral. 21. In the FAA-McDonnell Douglas DC-10 case, the author implies or asserts which of the following? a. The FAA immediately fixed the cargo door. b. The FAA told McDonnell Douglas to strengthen the cargo door and cabin floor; McDonnell Douglas eventually complied. c. The FAA was guilty of political favoritism and weak supervision toward McDonnell Douglas. d. none of the above 22. One common aspect of the various social sciences--history, economics, sociology, and political science--is that they all study a. relations among people. b. questions of power and authority. c. income disparities among social groups. d. all of the above 23. If American government approaches the economy with policies of noninterference, that implies a. an economic depression. b. radical change and upheaval. c. support for society's existing values. d. dissent from existing welfare programs. 24. A democracy where people rule indirectly is best described as a. representative. b. inclusive of all people. c. anarchic. d. conflict-free. 12

25. Checks and balances in the U.S. Constitution were needed to a. place limits on the power of future leaders. b. preserve federal power. c. allow the president unlimited power in foreign policy d. legitimize government power. TRUE-FALSE QUESTIONS 1. 2. 3. 4. 5. 6. 7. 8. 9. The "Guess Who's Coming to Dinner?" insert reveals that equality of wealth is a permanent attribute of U.S. democracy and capitalism. Authority is an inefficient form of power. Power and authority are not only central to politics, but to many other aspects of life. Political scientist Bernard Crick believed that violence, not conciliation, was the essence of democratic politics. Government places limits on the private distribution of values. Activism and apathy are both "political positions." Overlapping players and objectives characterize the political game in America. Politics in a democracy acts as a continually reacting "marketplace." Federal regulatory commissions have traditionally been closely tied to the economic interests they supervise.

10. More than 32 million Americans live below the poverty line. 11. Bill Gates is the founder of Microsoft. 13

12. Ethnic groups and foreign governments do influence American politics. ESSAY QUESTIONS 1. How is Harold Lasswell's definition of politics demonstrated by the "First Day of Class" story that begins the chapter? 2. Are political authorities necessary? Why or why not? 3. List and explain the basic functions of government in American society. Also, why is the game of American politics not like Monday Night Football? 4. How would the questions of an economist, historian, and psychologist differ from those of a political scientist in their studies of Microsoft? 5. How do governments affect the distribution of wealth in a society? Are certain methods considered more legitimate than others? Why? 6. Why does the author assert that "government does not monopolize the use of legitimate force, but it alone regulates its use? ANSWER KEY Chapter 1 Multiple Choice 1. 6. 11. 16. 21. B A D B C 2. 7. 12. 17. 22. D D D B A 3. 8. 13. 18. 23. A B B D C 14 4. 9. 14. 19. 24. C B C A A 5. 10. 15. 20. 25. D B D C D

True-False 1. 6. 11. F T T 2. 7. 12. F T T 3. 8. T T 4. 9. F T 5. 10. T T

15

CHAPTER TWO The Constitution: Rules of the Game


CHAPTER OVERVIEW This chapter discusses the official rules of the political game as they exist in the United States Constitution. The Constitution outlines the governments structure (the three branches of the federal government), restrictions on how the game can be played, legal protections for the losers, and the arenas in which play goes on (federalism). The chapter also explains the events surrounding the creation of the Constitution, the principles behind the document, how it evolved over time, and why constitutional rules continue to influence American political life. LEARNING OBJECTIVES After reading chapter 2, the student should be able to: 1. Describe the political events that led up to the ratification of the Articles of Confederation, as well as the strengths/weaknesses of those Articles. 2. Explain why the Constitution was a product of a series of compromises and what, specifically, the contents of those compromises were.

16

3. Summarize the important motives (Beards thesis) and political ideas of the Framers. 4. Describe the ratification struggle, noting the differences between the Federalists and Anti-Federalists and the issue of adding a Bill of Rights. 5. List and explain the significance of the four major constitutional principles--separation of powers/checks and balances, federalism, limited government, and judicial review. 6. Explain the process of amending the Constitution. 7. Elaborate on the reasons for the Constitutions durability, such as the stability of American society and the documents vagueness. 8. Explain the political relevance of and modern changes in federalism. OUTLINE I. The Constitution in the Game Analogy Introduction--A dialogue illustrating how students may detach the Constitution and the politicians who wrote it from the current perception of our national leaders. The Framers are viewed as patriots, while, today, elected officials are crooks. The Constitution contains the official rules of the American political game and establishes three major players: the president, Congress, and the Supreme Court. II. Background to the Constitution A. Introduction--Three sources for the Framers: 17

1. The English legal heritage, which limited the authority of government, illustrated by the Magna Carta and the writings of John Locke. 2. American models of colonial and state governments, which contained elements of the ideas of limited government, the consent of the community (Mayflower Compact), the separation of powers, the dominance of the legislature, and regular elections. 3. The experience with the Articles of Confederation (ratified in March of 1781), the failure of which underlined the need for a stronger central government that could impose taxes and have direct authority over individual citizens. B. The Articles of Confederation (1781-1789) 1. Shortcomings of the Articles included a weak league of friendship among the states, the requirement of unanimous approval by all thirteen states for amendments, no executive branch, no national system of courts, and no congressional power to impose taxes. 2. Strengths included the power to declare war, conduct foreign policy, coin money, manage a postal system, and oversee an army composed of state militias. A highly democratic feature was compulsory rotation in office. Other accomplishments included the start of a national bureaucracy and the passage of the Northwest Ordinance. 3. By 1787, the Confederation faced severe financial difficulties (devalued currency, trade wars among the states), foreign threats to American soil from the British, French, and Spanish, and the fear of mob rule and economic disruption (1786: Shays Rebellion). C. The Constitutional Convention (May 25 to September 17, 1787) 1. The Constitutional Convention met in Philadelphia, in strict secrecy. The delegates changed their original charge from a 18

revision of the Articles to the creation of a new government. 2. The Constitution was a product of compromises concerning the representation of large and small states in Congress (the Great Compromise, whereby representation in the House was determined by a states population, while equal representation in the Senate was accorded to all states); regulation of commerce (no tax on exports); and slaves being counted as three-fifths of a person (the slave trade was not banned until 1808). D. The Framers (55 delegates to the Constitutional Convention) 1. The Framers were politically experienced, well-educated nationalists, and, generally, members of the conservative propertied elite. They were wealthy planters, merchants, and lawyers. And many of them owned slaves. Small farmers, common workers, and liberals in the elite were not represented. Also, leaders who represented the poorer majority, such as Thomas Jefferson and Patrick Henry, did not attend the convention. 2. Convention debates were not between the haves and the have-nots, but rather among the haves from different regions. Motives Behind the Constitution 1. Charles Beard argued, in 1913, that the delegates were mainly concerned with creating a central government committed to honoring previous debts and protecting their own property holdings, and that the delegates did not favor democracy. 2. Critics of Beard have asserted that the delegates had varied motives, such as promoting economic development and raising a federal army that could protect the states from foreign aggressors. 19

E.

F.

3. The Framers public interest of nation-building coincided with their private interest of protecting their property. Federalists Versus Anti-Federalists--Ratification and the Bill of Rights 1. The Federalists favored a strong central government to curb the power of the common people and of the states. The Anti-Federalists favored a weaker central government in order to tip the balance of power toward the states. They desired a rigid system of separation of powers and effective checks and balances. The Constitution was the result of compromises between these two positions. 2. Conventions in nine states had to approve the Constitution. The Federalist Papers, authored by James Madison, Alexander Hamilton, and John Jay, stressed the advantages of national union and undoubtedly helped the ratification process, especially in the state of New York. 3. The Anti-Federalists insisted (successfully) on the addition of a Bill of Rights to the Constitution, arguing that these rights were sacred and universal for present and future generations of Americans. 4. Note the boxed insert, "Is the Constitution AntiDemocratic?", in which the late Thurgood Marshall observes that the Constitution excluded women and minorities. To Marshall, it was the Civil War that "created virtually a new Constitution using the Fourteenth Amendment to ensure the rights of all Americans."

III.

Four Major Constitutional Principles A. Introduction. The Constitution establishes the structure of government, distributes certain powers, and puts restraints on the government. In short, the Constitution both grants and limits governmental power.

20

B.

Separation of Powers and Checks and Balances. These principles place the powers of government in the care of separate parts and then mix together some of the powers to ensure the relative independence of the three branches (legislative, executive, judicial). Although never explicitly described in the Constitution, this philosophic principle was put into practice by the colonial governments. But while the branches of government are separate, in practice they share the overall power of government. Examples of checks and balances include the presidential veto, Senate confirmation of presidential nominations, and the courts ruling a law unconstitutional. Also, note Madison's observation (see boxed insert) that "if men were angels, no government would be necessary."

C. Federalism. This idea calls for authority to be distributed between a central government and the governments of the states. This leads to an inherent contradiction: Can the central government be supreme and state governments be independent? This contradiction was resolved historicallyby arms during the Civil Warin favor of the federal government. As the country grew in size and population, the need to solve problems (e.g., the environment and corporate regulation) moved beyond the capacities of the states. The nationalization of political issues led, in part, to the nationalization of solutions in areas like public education, a responsibility originally reserved to the states. Modern federalism appears far different from the original concept. Most nonmilitary services provided by government are supplied by local and state governments, in complex overlapping relationships with the federal government. The debate over modern federalism has resulted in liberals and conservatives battling over whether programs administered on a local level are closer to peoples lives and more efficiently 21

managed. By 1996, President Clinton and the Republican Congress worked together to shift more responsibility to local governments and reduced the federal role in a number of programs, including welfare. In 2001, President Bush sent legislation to Congress whereby the federal government set standards for student achievement backed by tests, but also let the states decide on how to implement them. So, federalism remains a flexible system for representing the varied interests of a large, diverse nation. D. Limited Government. This incorporates both civil liberties and civil rights, and ensures citizens rights against the government, as well as access to the government. Examples are the rights to vote, to dissent, and to an impartial jury. Judicial Review. A principle not specifically stated in the Constitution, but developed through judicial precedent, beginning with Marbury v. Madison, 1803. Its most vital function has been to protect the union by allowing the Court to review state and local laws. Although few federal laws have been struck down by the Court, hundreds of state and local laws have been held to violate the Constitution.

E.

IV. How is the Constitution Changed? A. Amendments. This is the least common method because of its difficulty. The usual procedure is proposal by two-thirds vote of each house and ratification by three-quarters of the state legislatures (normal deadline is seven years for passage). An exception is the Twenty-first Amendment, ratified by state conventions. There are 27 Amendments to the Constitution, with the 27th Amendment (pay raises for Congress cannot be granted until an election has occurred) being finally ratified in 1992. 22

B.

Judicial Interpretation. This is the most common method. The Supreme Court has reshaped the original document by interpreting vague constitutional phrases and allowing or disallowing practices by various political players. Supreme Court decisions have impacted economic regulation, segregation of the races, obscenity laws, and voting rights. Indeed, the Supreme Court is often called a permanent constitutional convention. Legislation. Congress has filled in the framework of the Constitution by establishing lower courts, the cabinet, executive boards and commissions, regulations, and services (e.g., Social Security). Custom. One of the vaguest but most significant methods, custom has created the role of political parties, party leadership in government, the congressional committee system, and the breakdown of the Electoral College. Customary usage has even changed some clear intentions of the Framers, such as in the setting of excessive bail. Custom now allows a president to enter armed conflict (e.g., Korea or Vietnam) without a declaration of war.

C.

D.

V.

Why Has The Constitution Survived? A. B. The stability of American society is the key reason. Social and political upheavals have all been handled within the same constitutional structure. The shortness of the document (only 7,000 words) and the ambiguity of constitutional language, which allows for different interpretations over time, have also assured survival.

VI. CASE STUDY: Federalism at 55 mph

23

The 1995 Congressional repeal of the national 55-mph speed limit would seem to be a clear victory for states rights. But a closer look at the Senate action reversing this unpopular measure reveals a comparable victory for federalism: the Senate also voted to keep federal limits on big trucks, to keep seat belt requirements, and to require tough new state laws against drinking and driving by minors. Thus, the popularity or unpopularity of certain safety measures proved as influential as either political philosophy in the actions taken by Congress. VII. Wrap-Up The question is whether the flexibility and vagueness of the Constitution makes it meaningless for governing the nation. Does the document merely serve the interests of those in power? All great historical documents have been differently applied by different people at different times. The Constitution stands as a symbol of a peoples ideals. In the final analysis, the rules of the American political game are guidelines and goals that depend upon each political generation for their application. KEY CHAPTER TERMS AND IDENTIFICATIONS Constitutional "limits" on the American political game May, 1787--Philadelphia and the Constitutional Convention Magna Carta (1215) John Locke Mayflower Compact (1620) Declaration of Independence Articles of Confederation (1781-1789); a type of "term limits" Northwest Ordinance Unicameral (one-house) legislature Continental Congress Shays Rebellion (1786) and fears of "mob rule" 24

"Revising" the Articles Washington, Hamilton, Madison, and Franklin The "absence" of Jefferson Great Compromise Three-Fifths Compromise Charles Beards An Economic Interpretation of the Constitution of the United States (1913) Public interests vs. private interests of the Framers Federalists vs. Anti-Federalists (Patrick Henry, George Mason) The Federalist Papers (James Madison, Alexander Hamilton, John Jay) Bill of Rights Separation of Powers/Checks and Balances Hofstadter's "harmonious system of mutual frustration" Federalism and the "marblecake" model President George W. Bush's "No Child Left Behind" education plan Limited government; a "political compact" based on rule of law Judicial review Marbury v. Madison Reserved powers to the states--Tenth Amendment States rights and "devolution of power" Exclusive powers to the national government Constitutional amendments and Article V Twenty-First Amendment (Prohibition Repeal)--state conventions The Constitution's "durability, flexibility, brevity" 27th Amendment or the Madison pay raise amendment TEACHING SUGGESTIONS 1. Do students have an idealistic view of the Framers of the Constitution? Is their view of modern politicians equally unrealistic, in the other direction?

25

2. Specifically compare and contrast the U.S. Constitution and the Articles of Confederation. Ask members of the class to explain why the differences were so important. 3. Read excerpts from the writings of the Anti-Federalists so that students can better understand the precise nature of their objections to the proposed Constitution. 4. Ask the class to speculate on how American history might have been changed if the Constitution had not been ratified. 5. Debate the following assertion in class: The Constitution was designed to be anti-democratic--to keep common people from having too much power. Be sure the class comments on the Thurgood Marshall material. 6. Imagine that the Constitution had been copied in another country. Compare the impact of Americas written Constitution on U.S. history with its impact on another countrys history. 7. Analyze how President George W. Bush's educational policies were related to patterns of federalism. SUGGESTED RESEARCH ASSIGNMENTS 1. Ask selected students to investigate why there have been calls for a new Constitutional Convention in recent years. What subjects would the new delegates consider? Also, what dangers might stem from a new convention? Finally, how would the modern media coverage change this event? 2. Is a written Constitution necessary for a democracy to exist? Great Britain does not have one, while some dictatorships have constitutions that promise individual rights against the state, but do nothing to allow 26

them. Ask students to consider, in writing or orally, which specific historical and political factors make constitutional rights relevant to a nations behavior. MULTIPLE-CHOICE QUESTIONS 1. In the opening dialogue, the students considered the Framers of the Constitution to be a. above politics. b. politicians just like modern ones. c. dead White males. d. neanderthals. The Magna Carta a. established the concept of a bicameral legislature. b. declared that the kings power was unlimited. c. included the idea of natural rights for the individual. d. established the principle of judicial review.

2.

3. The Mayflower Compact a. established the first colony on Plymouth Rock. b. settled a territorial dispute between the colonists and Native Americans. c. was the earliest American example of written rules of law resting on the consent of the community. d. was the peace treaty that ended the War of 1812. 4. The Articles of Confederation a. established a league of friendship among the states. b. required nine states of the thirteen to approve new amendments . c. did give the power of taxation to the Continental Congress. d. all of the above are true statements about the Articles

27

5.

The Great Compromise a. established representation based on population in the House of Representatives. b. created equal representation for all states in the Senate. c. solved a major disagreement between the large and small states. d. all of the above The members of the Constitutional Convention were generally a. wealthy landowners and merchants. b. average farmers selected at rural meetings. c. textile workers displaced by the Industrial Revolution. d. advocates for the huddled masses yearning to breathe free. Which of the following men did not attend the Constitutional Convention? a. Benjamin Franklin b. George Washington c. Patrick Henry d. Alexander Hamilton According to economic historian Charles Beard, the delegates to the Constitutional Convention were believers in a. improving their own economic interests. b. democracy. c. government not honoring private debts incurred under the Articles of Confederation. d. all of the above

6.

7.

8.

28

9.

Which of the following is a true statement about the Federalists and/or Anti-Federalists? a. Both factions distrusted the power of state governments. b. Federalists were pessimistic about human nature; Anti-Federalists were more optimistic about human nature. c. The Anti-Federalists preferred a non-elected judiciary and indirectly elected president. d. All of the above are true statements.

10. The Federalist Papers a. argued that a stronger national government threatened liberty. b. supported redistributing wealth through the federal government. c. reassured the public that the proposed president would be more like a governor than a king. d. has an ironic title because it represented the thinking of the AntiFederalists. 11. The following is not a major constitutional principle a. federalism b. separation of powers c. the principle of self-determination d. limited government 12. Which of the following is an example of checks and balances? a. a presidential veto b. the Senate confirmation of a presidential nomination c. the Congress refusing to appropriate money for an executive agency d. all of the above

29

13. Despite the principle of separation of powers, which of the following is an example of the presidents (the executive) legislative powers? a. the power to set tariffs on imported goods b. the power to recommend measures to Congress c. executive review of Supreme Court decisions d. the power to remove Cabinet Secretaries 14. The U.S. Constitution is roughly how long? a. 30,000 words b. 7,000 words c. 1,000 words d. 17,000 words 15. Why was the division of power among the three branches of government insufficient in itself to balance their powers? a. The Legislature, as the only initiator of laws, could dominate the other two. b. Congress has more members. c. Supreme Court judges are appointed for life. d. The President could become a dictator. 16. Under the system of federalism, a. only the federal government has exclusive powers. b. only the state government has exclusive powers. c. power is unlikely to be concentrated in one level of government. d. only state governments act directly upon the people. 17. President Clinton's welfare reform legislation did which of the following? a. The legislation reflected traditional Republican principles. b. It used block grants. c. It transferred control over welfare to the states. d. The legislation did all of the above.

30

18. Which of the following statements is true about judicial review? a. It was first established by the 1803 Marbury v. Madison Supreme Court ruling. b. It has been used more often in striking down federal laws than state and local laws. c. Both a and b are true. d. Both a and b are false. 19. Which of the following is a method of amending the Constitution? a. a two-thirds vote of each house of Congress and three-fourths of the state legislatures b. national referendum c. a majority of the Supreme Court and Congress, with the presidents signature d. three-fourths of state legislatures and the presidents signature 20. All of the following are examples of how custom has changed the Constitution except a. courts setting high bail b. presidents entering foreign conflicts c. courts prohibiting legal segregation of the races d. the breakdown of the Electoral College 21. Which of the following is an example of custom becoming law? a. the 1st Amendment outlawing the abridgement of free speech b. the 21st Amendment, which repealed the prohibition of alcohol c. the 22nd Amendment limiting a president to two terms d. the 19th Amendment, which gave women the right to vote 22. The permanent constitutional convention refers to the a. Supreme Court. b. federal bureaucracy. c. fifty state legislatures. d. Congress. 31

23. The 27th Amendment is concerned with a. changing the voting age to 18. b. the due process clause. c. pay raises for members of Congress. d. illegal search and seizure. 24. All of the following are sources of strength for the Constitution except a. its shortness b. the vagueness of its language c. its inflexibility d. the relative stability of American society 25. Realistic application of constitutional principles depends on a. the hypocrisy of politicians. b. the absolutist language of the document. c. an understanding of the motives of the Framers. d. current political relationships. 26. All the following are examples of the anti-democratic nature of the original Constitution, except a. the Electoral College b. the election of U.S. Senators c. the Tenth Amendment d. the presidential veto 27. Writing as a political scientist in 1908, Woodrow Wilson argued that a. federalism is an ever-changing situation depending on political and economic development. b. the president had gained too much power since the Lincoln administration. c. industrialization would lead to the elimination of federalism. d. the League of Nations would secure peace throughout the century.

32

28. Public education involves the federal government in the following way: a. aid programs help equalize state funding. b. forbidding racial discrimination. c. overseeing local school boards. d. a and b 29. Which former Supreme Court Justice argued that the original Constitution was undemocratic? a. Earl Warren b. Thurgood Marshall c. William Howard Taft d. William Douglas 30. Federal speed limits have been connected with all the following political concerns except a. highway safety b. federalism c. crime d. energy conservation TRUE-FALSE 1. 2. 3. Most of the colonies were ruled by parliamentary systems, like the government in Great Britain, and mandated by the Magna Carta. Thomas Jefferson and Patrick Henry were instrumental in getting the Great Compromise passed at the Constitutional Convention. The Great Compromise meant that the equality of the states was assured in the Senate, while large/small state differences were maintained in the House.

33

4. 5. 6. 7.

The Anti-Federalists supported strong state governments, because they believed that states would be close to the popular will. The Bill of Rights basically limits governmental power. A block grant is a wide group of civil rights that are granted to new citizens and immigrants. Public education represents marblecake federalism, because local, state, and federal levels of government are all involved in its operation. All amendments have been proposed by Congress. The practice of judicial review is not mentioned in the Constitution.

8. 9.

10. By removing federally mandated speed limits, the federal government devolved power regarding all highway safety issues. ESSAY QUESTIONS 1. What were the factors convincing many propertied elite members in the 1780s that a stronger national government was necessary? 2. Explain what the author means when he asserts that the debates at the Constitutional Convention were between the haves and the haves, over their regional interests, not between the haves and the havenots. 3. What are some of the drawbacks of checks and balances and separation of powers? As modernization and technology become more widespread, can the federal government still operate effectively? 4. Why are there debates over "Modern Federalism"? 34

5. Using the amendment process and federalism as examples, explain how the Constitution influences current politics. ANSWER KEY Chapter 2 Multiple Choice 1. B 2. C 6. A 7. C 11. C 12. D 16. C 17. D 21. C 22. A 26. C 27. A True-False 1. F 6. F 2. F 7. T 3. C 8. A 13. B 18. A 23. C 28. D 3. T 8. T 4. D 9. B 14. B 19. A 24. C 29. B 4. T 9. T 5. D 10. C 15. A 20. C 25. D 30. C 5. T 10. T

35

CHAPTER THREE The Executive Branch: The President and the Bureaucracy

CHAPTER OVERVIEW This chapter stresses the contrasting public perceptions of the presidency, its historic evolution, various types of presidents, and the several hats a president wears, such as Chief Diplomat, Party Leader, and Commander-in-chief. The chapter outlines the history of the presidency, the constitutional role of the president, and his influence over public opinion. The presidential relationship to the executive bureaucracy is also explored, following the theme of the bureaucracy as both a tool of and a limit on presidential power. Finally, the case study on President George W. Bush's response to September 11 reveals his leadership abilities, along with the way a president rallies the nation in a time of crisis. LEARNING OBJECTIVES After reading chapter 3, the student should be able to: 1. Explain how Constitutional provisions affect the performance and responsibilities of the president. 2. Review those key events that shaped the evolution of the presidency.

36

3. Categorize presidents according to the Buchanan, Lincoln, and Eisenhower approaches, as well as the James Barber typology. 4. Examine and describe the six overlapping presidential hats. 5. Explain the significance of the Executive Office of the President, the Cabinet departments, the Executive Agencies, and the Regulatory Agencies. 6. Describe the relationship between the president and the bureaucracy, the problems of the bureaucracy, and its policy-making powers and limits. 7. Explain how a presidents influence over public opinion can influence Washingtons policy agenda. 8. Explain why President George W. Bush has been labeled the "Imperial Delegator," and also explain his general style/policies. 9. Review the key points made in the chapter study dealing with the terrorist attacks of 9/11 and their aftermath. 10. Explain the origins and purpose of the U.S. Civil Service. OUTLINE I. Introduction. The president is the superstar in the political game. He is the only official elected by the entire country. Yet, the public view of him as the nations leader has historically led to unrealistic expectations and crushing disappointments. Clinton was one example of this. Yet, by contrast, George W. Bush's presidential effectiveness has surprised those who underestimated his overall abilities. 37

II.

The President and the Constitution. The Constitution grants few specific powers and duties to the president, but vague phrases have been interpreted to greatly enlarge them. Presidential qualifications for office, the concept of a lame duck, and the functions (note the disputed 2000 election) of the Electoral College (270 electoral votes are required to be elected) are analyzed (after 2000, there was talk of abolishing the EC, but nothing happened). The duties of the vicepresident are outlined (presides over the Senate; succeeds the president if death or disability occurs) and the political importance of that office as a step to the presidency is discussed. The new vicepresidency is mentioned where vice-presidents like Walter Mondale, George Bush, Al Gore, and Dick Cheney have played roles of deputy president. Cheney probably became the most powerful vice president in the nation's history under George W. Bush--he was often referred to as the "prime minister" in the White House. History of the Presidency. The role and influence of the president have expanded far beyond the constitutional conception of a gentleman-aristocrat who would stand above politics. Most members of the Constitutional Convention in 1787 did not even see a political role for the president. Congress was meant to stand supreme, but the responses of presidents to crises have increased the power of the presidency. Washington used residual powers (powers not spelled out in the Constitution, but necessary to carry out other responsibilities) in putting down the Whiskey Rebellion; Jefferson weakened congressional control of foreign affairs by negotiating the Louisiana Purchase; and Abraham Lincoln expanded the role of the president in wartime. Franklin D. Roosevelts response to the Depression and World War II firmly established the modern presidency and its strong leadership patterns.

III.

IV. Types of Presidents A. Buchanan Presidents take a custodial view of their responsibilities, remain aloof from politics, and exercise only 38

those powers directly stated in the Constitution. Harding, Coolidge, and Hoover are examples. B. Lincoln Presidents are active politicians who take a strong leadership role in crises. They view the presidency as a stewardship--that is, the only limits on presidential action are those expressly stated in the Constitution. But they do not interpret the Constitution narrowly. Jackson, Theodore Roosevelt, FDR, Truman, Johnson, and Nixon are examples. Eisenhower Presidents combine the two types by linking the political inactivity of the Buchanan model with an active role as the chief delegate of the people, who organizes and represents a national consensus. Recent presidents have all been active, but have mixed these styles with varying emphases. Some observers saw parallels to Eisenhower in George Bushs style of leadership. This is sometimes called the "hidden-hand leadership." Modern Presidents have leaned toward political activism, including Lyndon Johnson and Jimmy Carter. Some have done better in foreign policy than domestic policy (George Bush). President Clinton, in the beginning, initiated an activist administration by proposing a number of new programs, including health care, with mixed success. After the Republican takeover of Congress in 1994, Clintons ambitious legislative program stalled, so he compromised with the Republicans on some issues (e.g., welfare), while positioning himself as a bulwark against congressional extremism. George W. Bush, the first president to lose the popular vote since 1888, adopted a detached attitude, delegating responsibility to experienced administrators. A Psychological Approach. Political scientist James Barber has combined the presidents style (his ability to interact) with his 39

C.

D.

E.

character (essential nature or personal qualities) to come up with four categories of presidents: active-positive, active-negative, passive-positive, and passive-negative. He labels John Kennedy and Jimmy Carter as active-positive. President Eisenhowers tendency to withdraw from conflict places him in the passivenegative category. Unfortunately, these categories neither reflect the totality of a presidents career nor consider other political forces influencing the presidency (see "Presidential Mama's Boys"). V. Presidential Hats The presidents six overlapping hats/roles have increased in importance over time. A. Chief of State. This symbolic and ceremonial function is combined with the real powers of the chief executive, thus enhancing the authority of the president and the influence of his party. In this role, many people see the president as a symbol of the nation. B. Chief Executive. In theory, the president is in complete control of the huge federal bureaucracy, which employs some 1.8 million civilians, spends over $1.7 trillion a year, and ranks as the largest administrative organization in the world. Criticism of this bureaucracy is widespread, and most modern presidents have made its reduction a goal of their administration. C. Chief Diplomat. There are fewer checks on the presidents conduct of foreign affairs than of domestic affairs. The president establishes relations with foreign governments, appoints ambassadors, and negotiates and signs treaties that take effect with the consent of two-thirds of the Senate. The Cold War extended the presidents diplomatic responsibility into nearly all areas of society. Investigations of covert actions have revealed the dangers of this trend. The Senates power to ratify or reject treaties has been weakened by the use of executive agreements 40

that require no Senate approval or even public disclosure. Congressional attempts to limit the use of executive agreements have failed, but Congress can refuse to appropriate funds to carry out the agreements. D. Commander-in-chief. The president was made commander-inchief in order to maintain civilian control of the military, but in practice, his authority is represented by the secretary of defense and delegated to the leaders of the military. This role is an important adjunct to that of chief diplomat, because of the presidents power to use troops at home and abroad, and because of the size and importance of the defense establishment. Although the Constitution gives Congress the power to declare war, presidents have initiated large-scale military involvement of the U.S. in both Korea and Vietnam. Reaction to this kind of involvement culminated in the 1973 bill (The War Powers Act), limiting the presidents ability to commit troops abroad. The laws effectiveness remains uncertain. President Bush ignored the Act in the invasion of Panama and considered it unconstitutional in the 1991 Iraq War. President Clinton also avoided congressional approval when he put military units in harms way in Bosnia and Kosovo. Chief Legislator. It was not until the 20th century that presidents regularly and actively used their constitutional right to initiate legislation in Congress. Presidents often try to control the national agenda by identifying important political issues during the annual State of the Union address or during other statements. The president may get support for a bill by using either informal tactics--such as exclusion from social functions and threats to block a members special legislation--or his constitutional right to veto legislation. The threat of a veto is often used to force a compromise. The president has been traditionally limited to vetoing the whole bill, but, in 1996, Congress gave the president a limited item veto, the power to veto just parts of the bill he 41

E.

dislikes. But one year later, the Supreme Court ruled that the item veto was unconstitutional. F. Party Leader. The presidents duties as the head of his political party are to choose a vice-president, to distribute offices, to try to fulfill the party platform, and to serve as chief fund-raiser and campaigner. His control is limited, however, by the decentralized nature of American politics. Ultimately, the degree of presidential involvement in party politics is up to him. George W. Bush's informal power over GOP congressional candidates was reflected in the 2002 elections.

VI. The President and the Public A. The President as a National Symbol. The presidents visibility, public exposure, and symbolic importance give him great influence in the political game, but this influence can become a liability if the exposure is unfavorable. The media plays a pivotal and often negative role. This is seen in the ups and downs experienced by our two most recent presidents, Bush and Clinton.

VII. The Federal Bureaucracy A. The Executive Office of the President. This department was established in 1939 to help the president manage the bureaucracy and has since grown into a major bureaucracy itself, employing over 1,400 people in eight agencies. The White House staff, whose members are responsible solely to the president, has recently grown in authority at the expense of the cabinet. (Andrew Card was the White House chief of staff for George W. Bush. Note that Bush tends to run his staff like the CEO of a large corporation.) The National Security Council, established to help coordinate the departments of Defense and State, varies in importance with its use by individual presidents. 42

Under Condoleeza Rice, Bush's NSC advisor, the NSC "became smaller and less public." Other councils include the National Economic Council, the Office of Management and Budget, and the Council of Economic Advisors. B. The Cabinet Departments. Their expansion from three to fifteen has been largely due to the growth of problems that people want the federal government to address. A good example of this expansion was the creation of the Department of Homeland Security, the newest cabinet department, created to deal with the terrorist threat after 9/11. The DHS has 22 agencies and 170,000 employees. The cabinet has no real power, as a body independent of the president. Although many presidents enter office promising to give the cabinet more power, it hardly ever turns out that way. The degree of control that Cabinet officers exercise over their individual departments varies. The Executive Agencies. The Office of Personnel Management and the National Aeronautics and Space Administration are like the cabinet departments, but are not important enough to be included in the cabinet. Some agencies are government corporations, such as the Tennessee Valley Authority and the Post Office, which are semi-independent units performing business functions.

C.

D. The Regulatory Commissions. Supposedly bipartisan, these relatively independent agencies of the government were established in order to regulate certain parts of the economy (e.g., interstate trade) and to keep them responsive to public interests. Although the president appoints the members of the commissions and chooses who chairs them, the commissions are relatively independent of all branches of government. However, they are often accused of being more responsive to the groups they regulate than to the public. Note the text example of the 43

Securities and Exchange Commission and its relationship to Enron, in 2002. VIII. The Problems of Bureaucracy A. Introduction. The size and complexity of large bureaucracies makes it difficult to define responsibility, thus inhibiting public scrutiny and control. The necessity for expertise cuts down bureaucrats appreciation of a broader view of the public interest. Various remedies have been proposed, such as consumer pressure groups, to act as watchdogs and localize control. Rise of the Civil Service. The establishment of the civil service, a major 19th century reform that emerged after a disappointed office seeker assassinated President James Garfield, has improved the quality of government employees and added stability to government. The spoils system, which permitted elected officials to fill government positions, has been restrained by the professionalization of the bureaucracy. Today, the president only fills about 5,000 patronage jobs. Bureaucrats as Policy Makers. The ideal model of separating policy and administration is incomplete and naive. A bureaucrat is rarely politically neutral; he/she does not merely implement policies initiated by others. Bureaucracies are involved in policy-making because they exercise legislative, judicial, and executive power. Thus, for example, the IRS--Internal Revenue Service--holds hearings on tax cases and makes judicial findings.

B.

C.

D. The President and the Bureaucracy. The bureaucracy is a major source of information and power for the president, but its unwieldy structure requires controls and may often work against him. Keeping control over two million employees is a full-time 44

job in itself. Members of the bureaucracy may work to protect their own interests or their departments when threatened by budget cuts. They may ignore the presidents demands and delay or sabotage his agenda. To gain the support of his bureaucracy, the president must bargain and persuade. IX. Case Study: September 11th--A President's Trial by Fire" The case study deals with the way President George W. Bush responded to the horrific 9/11 terrorist attacks upon America. The President rallied the nation and promised to find those who were responsible for those attacks. He prepared the nation for a new kind of war, comforted the American people, and was a visible "democratic priest-king." In his 9/20 speech before Congress, GWB dramatically delineated the lengthy and difficult challenges confronting the country.

KEY CHAPTER TERMS AND IDENTIFICATIONS Democratic priest-king Article II of the Constitution George W. Bush--"The Imperial Delegator": Twenty-Second Amendment Lame-duck Tilden-Hayes 1876 election Electoral College and its role in 2000 270 electoral votes New vice-presidency--Dick Cheney as example Whiskey Rebellion Residual or inherent powers "pattern of crisis leadership" Louisiana Purchase (Thomas Jefferson) Buchanan, Lincoln, Eisenhower Presidents "hidden-hand leadership"; stewardship presidency FDR's New Deal 45

Custodial view of presidential powers Clinton-a "New Democrat" George Bush (Sr.) and Kuwait Barbers active/passive style; positive/negative characters Active-Positive (Kennedy) Active-Negative (Nixon) Passive-Positive (Reagan) Passive-Negative (Eisenhower) Chief of State; Chief Executive Chief Diplomat Executive Agreements Commander-in-Chief Civilian supremacy War Powers Act of 1973 Chief Legislator--The "National Agenda" State of the Union address Item veto--ruled unconstitutional in Clinton v. City of New York Pocket veto Party Leader Party platform White House "spin" (Monica Lewinsky scandal) Theodore Roosevelt--the presidency as a "bully pulpit" bureaucrat EOP--WHO, NSC, OMB, CEA White House Chief of Staff (Andrew Card) George W. Bush and delegation of authority Karl Rove Cabinet departments Department of Homeland Security National Security Council; Condoleeza Rice Secretary of Defense Donald Rumsfeld Impeachment First Ladies--Hillary Clinton; Lady Bird Johnson Executive Agencies--OPM, NASA, CIA Tennessee Valley Authority 46

Regulatory Commissions--ICC, FCC Securities and Exchange Commission and Enron FBI/CIA roles after 9/11 Spoils System Pendleton Act or Civil Service Reform Act (1883) Public administration Bureaucratic efficiency Department of Homeland Security TEACHING SUGGESTIONS 1. Ask the members of the class to define presidential greatness. List the characteristics of a great president on the blackboard. Then ask the class to consider whether any or all of these characteristics apply to any recent presidents. Include the performance of George W. Bush after 9/11. 2. Run a videotape of a recent presidential press conference and/or major speech. Ask members of the class to judge how effectively the current president uses television to reach the public, as well as the quality of his relationship with the press. How can a hostile media influence the public image of a president and his administration? 3. Review how the actions of regulatory commissions affect the quality of American life and how they impact the individual citizen. You may wish to assign a student or two to investigate the actions of a particular commission during the past year or two. 4. Lecture on why the effectiveness of the 1973 War Powers Act has been questioned. 5. Review the disputed presidential election of 2000. Ask the students whether they believe the Electoral College should be abolished, considering what transpired in that election. 47

SUGGESTED RESEARCH ASSIGNMENTS 1. Form a team of students to investigate how one applies for a civil service job with the federal government. The students can report their findings to the class as a whole. They also could provide copies of application forms, as well as fundamental civil service rules and regulations. 2. The popularity of presidents rises and falls over time. Ask students to chart what the public approval rating of the current president was when he entered office and what it is now. Have the students write a short paper accounting for its decline, rise, or stability. See if they can remember what the expectations were when he entered office. What explains these expectations? 3. Ask a student to investigate a recent example of the bureaucracy opposing a presidential directive. The student can report in class why the agency/department worked to stop the presidential action, how they did it, and discuss how the president responded. What were the roles of Congress and the media during the disagreement? 4. Ask a student to investigate how the states and the federal government coped with the issue of electoral reform after the disputed recounts and controversial ballots from the state of Florida in election 2000. 5. Ask a team of students to evaluate the role of vice president Dick Cheney in the administration of George W. Bush. What were Cheney's key policy roles? How did he influence the president's decisions, especially after 9/11? 6. How has Hollywood viewed the presidency? Have film producers/directors helped the American public to understand the presidency from both personal and institutional perspectives? Why or why not? 48

7. Investigate the relationship between the SEC and Enron. 8. Visit the Department of Homeland Security web site. What can one find regarding the Department's mission and composition? MULTIPLE-CHOICE QUESTIONS 1. Thomas Langston believes the publics expectations of presidents are frequently a. cynically manipulated. b. realistic. c. inflated and disappointed. d. a and b Which of the following is not a constitutional requirement for serving as president? a. being a resident of the U.S. for 14 years b. being a member of Congress for at least one term c. being a native-born citizen d. being at least 35 years old Which of the following is not true about the Electoral College? a. except for Maine and Nebraska, it works on a winner take all system b. a candidate needs 270 electoral votes to become president c. if no candidate receives a majority of the Electoral College, the election is decided by a majority vote in the House of Representatives, with each state delegation casting one vote d. an elector must vote for the candidate that he pledged to support

2.

3.

49

4.

The Executive Office includes all of the following except the a. Council of Economic Advisors b. Office of Management and Budget c. National Economic Council d. State Department Who said that the vice-presidency was not worth a pitcher of warm spit? a. Harry S. Truman b. Al Gore c. Gary Wasserman d. John Nance Garner George Washingtons action in putting down the Whiskey Rebellion a. was endorsed by the Electoral College. b. is an example of residual powers of the presidency. c. was considered unconstitutional at the time. d. shows how weak the Presidency was before 1803. Buchanan presidents a. are favored by political commentator Patrick J. Buchanan. b. are considered to be the greatest of all presidents. c. merge the active and administrative parts of executive power. d. view their presidential terms as primarily administrative. Franklin D. Roosevelt is called the first modern president because he a. ran for three terms. b. attempted to pack the Supreme Court. c. was the first president to involve the United States in a world war. d. established strong leadership patterns in both domestic and foreign policy, and in creating the executive bureaucracy.

5.

6.

7.

8.

50

9.

Executive agreements a. unlike treaties, do not require Senate approval. b. have decreased in the 20th century. c. require approval by both houses of Congress. d. are agreements among officials who head the various agencies comprising the White House Office.

10. The role of Chief of State is most like a. the Prime Minister of England. b. the Queen of England. c. the Secretary of State. d. the Chief Justice of the Supreme Court. 11. According to some critics, the nine-to-five president was a. Lyndon Johnson. b. Jimmy Carter. c. Ronald Reagan. d. George Bush. 12. Which of the following presidents is incorrectly matched with one of Barbers categories of presidential personality? a. Richard Nixon: Active-Negative b. John F. Kennedy: Active-Negative c. Ronald Reagan: Passive-Positive d. Dwight D. Eisenhower: Passive-Negative 13. In which case did the U.S. Congress declare war? a. the Vietnam War b. the Kosovo conflict c. the Persian Gulf War d. World War II

51

14. Which of the following is a true statement about the pocket veto? a. it is another name for a regular veto b. Congress has an opportunity to override the pocket veto c. it operates like an item veto because the 10 day requirement is also involved d. the pocket veto is never overridden by a congressional vote 15. Which of the following is true about the Line Item Veto? a. it was passed by a Republican Congress as part of the Contract With America b. it could only be used on money bills c. it was struck down as unconstitutional by the Supreme Court d. all of the above 16. Regarding the Cabinet Departments, which of the following is a true statement? a. Currently, there are 12 Cabinet Departments. b. The EPA was recently elevated to Cabinet-status. c. Cabinet officers can be dismissed by the president. d. all of the above are true statements 17. The Executive Office of the President a. was established in Article II of the Constitution. b. was established this century to assist the president in managing the bureaucracy. c. was an agency that Ronald Reagan tried to abolish when he assumed the presidency. d. none of the above 18. The National Security Council a. was abolished at the end of the Cold War. b. works to protect presidents from assassination. c. coordinates American military and foreign policies . d. was created by George W. Bush in 2001 after 9/11. 52

19. Cabinet secretaries a. serve two-year terms that correspond to the House of Representatives. b. hold office as long as the president wishes. c. are confirmed by the Senate for up to six years. d. participate in all facets of government. 20. The Civil Service system has a. basically undermined the spoils system. b. weakened presidential controls over the bureaucracy. c. both a and b are correct d. both a and b are incorrect 21. Which of the following is a government corporation? a. United Parcel Service b. U.S. Postal Service c. U.S. Army d. Federal Reserve Bank 22. The problems of government bureaucracy could best be summarized as inefficiency due to the a. capitalistic ideology of American government. b. overly broad interests of most bureaucracies. c. size and complexity of bureaucracies. d. unqualified staffs that make up the bureaucracy. 23. The War Powers Act a. limited the presidents ability to use troops overseas without Congressional approval. b. has largely been ignored by recent presidents. c. was passed over Richard Nixons veto. d. all of the above

53

24. The Civil Service Reform Act was passed by Congress because a. the public cried out against governmental inefficiency. b. the president had too much power over his cabinet. c. a disappointed office seeker assassinated the president. d. the Congress wanted to legalize the spoils system. 25. The chapter box study shows that President Clinton's powers were limited on the issue of a. tax cuts. b. social security. c. welfare reform. d. anti-tobacco policies. 26. The new vice-presidency a. delegates more power in the administration to the vice-president . b. suggests the new role the First Lady plays in politics. c. is the term given to a strong Chief of Staff. d. is best demonstrated by FDR's vice president, John Nance Garner. 27. Abraham Lincoln expanded the power of the presidency by a. spending money without congressional approval. b. creating the Department of War. c. signing the Louisiana Purchase. d. all of the above 28. After his defeat for re-election, President Bush was a. ignored by the Republican party. b. a lame duck. c. no longer treated as the legal president. d. more respected by his successor.

54

29. President George W. Bush became a "democratic priest-king" after a. proposing his education program. b. September 11, 2001. c. proposing his tax cut. d. winning a disputed 2000 election. 30. Which of the following is not part of the job description of the modern president? a. Chief Justice b. Chief Legislator c. Chief Diplomat d. Chief of State TRUE-FALSE QUESTIONS 1. 2. 3. 4. 5. 6. 7. The president and vice-president are the only public officials elected by the entire nation. The War Powers Act has repeatedly limited presidential powers. The line item veto was declared unconstitutional by the Supreme Court. Going Public is an important way that the bureaucracy stops presidential action. Some observers believe that President Eisenhower practiced a hidden-hand style of presidential leadership. Lame duck status creates sympathy for out-going presidents, thereby making them more popular than when they served. President George W. Bush prefers to delegate responsibility to his advisers. 55

8. 9.

The Department of Homeland Security was created during the waning days of the Clinton presidency. Nameless, faceless bureaucrats only do the job they are asked to do. They never engage in politics or policy-making.

10. In 2000, George W. Bush lost the popular vote, but won the electoral vote. ESSAY QUESTIONS 1. If most members of the Constitutional Convention did not see a political role for the president, what, according to the Constitution, did they see their president doing? 2. Cite some basic reasons why the powers of the presidency have expanded, especially in the modern era. How did the presidential powers of George W. Bush expand after 9/11? 3. What are the six roles of the president? List and explain the significance of each of the roles. 4. Why do you think so much is expected of a president? Are presidents partially to blame? Is the media? The public? Why is it likely that those expectations will not all be met? 5. Why do presidents usually enter office claiming to support cabinet government? What are the advantages? What are the disadvantages? Why do they change? 6. How and why can First Ladies influence their presidential husbands?

56

7. What are the major problems facing bureaucracies? Second, what lessons about bureaucratic behavior can be learned from President Carter's experience with the White House mouse? 8. How did September 11th and its aftermath challenge President Bush's leadership abilities? 9. How and why is the bureaucracy both a supporter for the president and a restraint upon his actions? 10. What is "White House Spin"? Cite an example. Chapter 3 Multiple Choice 1. C 2. B 6. B 7. D 11. C 12. B 16. C 17. B 21. B 22. C 26. A 27. A True-False 1. T 6. F 2. F 7. T 3. D 8. D 13. D 18. C 23. D 28. B 3. T 8. F 4. D 9. A 14. D 19. B 24. C 29. B 4. F 9. F 5. D 10. B 15. D 20. C 25. D 30. A 5. T 10. T

57

CHAPTER FOUR The Legislative Branch: Congress

CHAPTER OVERVIEW This chapter begins with the constitutional expectation that Congress would be the democratic centerpiece of the federal government. Things havent turned out that way, but Congress importance and complexity is brought out in discussions of its election rules, size, composition, and structure. The chapter reviews the importance of party control, the decline of House Speaker power, as exemplified by the transition from Gingrich to Hastert, the all-important budget process, and how a bill becomes a law. In addition, such important and related topics as the congressional committee system, the role of seniority, and the vital non-legislative functions such as investigations of the executive branch are stressed. The chapter concludes with a detailed case study of the congressional struggle to pass campaign finance reform. LEARNING OBJECTIVES After reading chapter 4, the student should be able to: 1. Evaluate the difficulties legislators face in trying to reconcile local and national interests. 2. Explain the prominent background characteristics of members of Congress. 58

3. Define and explain the significance of malapportionment, gerrymandering, term limits, seniority, standing committees, conference committees, subcommittees, select and joint committees, senatorial courtesy, legislative oversight, and the filibuster, among others. 4. Detail the various stages of the legislative process, including committee debate, floor votes, role of party leadership, consideration by both houses, and the veto process. 5. Explain the reasoning behind the organization of both the House and Senate. 6. Review the congressional budget process. 7. Explain changes in congressional procedures created by recent Republican majorities in Congress. 8. Explain the significant points made in the chapter's Case Study on "Campaign Finance Reform." OUTLINE I. Introduction - Congress in the Constitution To keep the legislature dominant, the authors of the Constitution gave the majority of powers and duties of government lawmaking to Congress. Congress can levy taxes, borrow money, raise armies, declare war, set up the federal judiciary, regulate commerce, and coin money. Congress also controls presidential power, by having the power to declare war and raise an army, to approve treaties, to create administrative offices, to override the presidents veto, and to impeach executive and judicial officers. Indeed, throughout the nineteenth century, Congress was the major player in shaping the nations policies. But despite these controls, the executive branch has grown in power at 59

the expense of Congress. Even with narrow Republican majorities in both Houses, in 2003, the two chambers often seemed locked in a conflicted embrace. Still, the legislature is vital to the political game. II. Makeup of the Senate and House A. Introduction. The basic facts about the size and composition of the two houses are given. The Congress is bicameral, with 435 members in the House (based on population) and 100 members in the Senate (2 per state). The average representative now serves over 647,000 constituents. In the first House, each member represented approximately 50,000 citizens. B. The Role of the Legislator. Legislators represent both national and local interests, making their role difficult, since these interests are often incompatible. But for most congressmen, constituents opinions must be heeded if they wish to win re-election. And among the most vital work of a congressman is caseworksolving constituents problems with the government (assisting veterans, helping with overdue social security benefits). C. Who are the Legislators? Qualifications for membership are given. They include the 25 and 30 years of age requirements for House and Senate members, respectively. Senators serve six-year terms, while Representatives serve only two years, before running for re-election. Congress is overwhelmingly white, male, Protestant, and dominated by the upper-middle-class professions of law, business, and banking. Women, blacks, and other minorities have been typically underrepresented, a situation that has changed in recent congresses. Senators were originally elected by the state legislatures to ensure less popular control, but, since 1913 (with the passage of the Seventeenth Amendment), they have been elected directly. An important problem facing Congress is careerism among legislators who spend their careers in 60

Congress. The attempt by states to pass term limits has been ruled unconstitutional by the Supreme Court, in 1995 (U.S. Term Limits v. Thornton). D. Malapportionment and Reapportionment. Malapportionment is a political tactic that allows state officials to strengthen the position of their party by drawing districts in which their opponents will be underrepresented. Gerrymandering is another device used to enhance a partys position in a district, either by packing (designing a safe district of solid supporters) or by cracking (splitting up the supporters of opponents). Racial gerrymandering became more common after the 1990 census. Malapportionment abuses have been largely corrected by the 1962 Supreme Court ruling that required districts to be equal in population. Recent Court decisions have also limited the role of race in drawing districts. But politics remains vital in the drawing of districts, as seen in the conflicts over population shifts to the West and South from the Midwest and Northeast, following the 2000 census. After 1990, New York lost three house seats, and California gained seven seats. After the 2000 census, states in the northeast again lost seats, while the so-called Sunbelt gained seats in Congress. Florida, Georgia, Arizona, and Texas gained two congressional seats, while New York and Pennsylvania lost two apiece. Several Midwestern and northern states lost one each, while a number of southern and southwestern states gained one. After 2000, redistricting helped Republicans to gain seven seats in the House. In addition, recredistricting helped incumbents. E. Organization of the House. The majority party chooses officers, controls debate, selects committee chairmen, and holds a majority on every committee. The Speaker of the House has considerable power, and Dennis Hastert is now the speaker of the majority party Republicans. He is assisted by the majority leader and majority whip in coordinating the partys position. The minority party has the same officers, minus the Speaker. Each party has a caucus, composed of all the members of that party in the House, which provides a fairly 61

unified following for party leaders. The Republicans choose a Steering Committee that functions as an executive committee for the caucus. After the 1994 election, the Democrats divided their Steering Committee into a Steering Panel, which nominates committee members, and a Policy Committee, which studies issues and writes bills. Nancy Pelosi is the Democrats minority leader. F. Organization of the Senate. As president of the Senate, the vice president votes in case of a tie and has some procedural power. The real leader of the Senate is the Senate Majority Leader, who schedules debates, assigns bills to committees, and coordinates party policy. Party positions and strategies are coordinated by the majority and minority leaders and their whips. The Senate, a smaller body, operates more informally than the House and is not as tightly controlled by its leadership. The House and Senate, in 2003, were controlled by the Republicans, given the GOP's gains in the 2002 midterm elections. Trent Lott was the Majority leader, and Tom Daschle of South Dakota became the Minority leader (Lott was later deposed and replaced by Bill Frist). The Republicans have a Committee on Committees, responsible for the assignment of Members to committees, as well as a Policy Committee that charts legislative strategy. Senate Democrats are organized similarly. III. How Congress Works (the Committee System) Legislation can be introduced in either house, except for moneyraising and appropriations bills, which must originate in the House. The two Houses act separately in drawing up a bill, but the language of the bills they pass must be identical before final passage. Due to the number and complexity of issues before Congress, bills are immediately assigned to a committee and must be approved there before being presented to the whole body. About 20,000 bills are introduced in Congress each year, but 80-90

62

percent of all bills die in committee, and only 5 percent of all bills become law. A. VHow Committees Work. Much of the work of Congress takes place in committees. Floor debate is little more than a formality designed to make a public record. There are four types of committees: 1. Standing committees are the basic working units of Congress to which all prospective legislation must be assigned. There are nineteen in the House and seventeen in the Senate; these are further divided into subcommittees, which have been dramatically reduced by the Republicans. In general, the recent Congress has reduced the power of committees by increasing the influence of the party leadership. However, before any bill is sent to the floor for consideration, it must still be approved by the committee covering that subject. 2. Conference committees are temporary bodies composed of senators and representatives, created to work out differences between House and Senate versions of one bill. These temporary committees bargain until a compromise is reached and legislation is sent to both houses with identical language. Once a compromise is reached, the committee disbands. Typically, senior members of the relevant committees serve on conference committees. 3. Select committees are set up to do specific, temporary jobs, such as investigations. The Senate Special Committee on Whitewater, formed to investigate any improper activities by President Clinton in an Arkansas real estate project, is an example. 4. Joint committees are permanent bodies with members from both houses, whose purpose is to coordinate policy and prevent duplication on routine matters. An example is the Joint Economic Committee. a. Committee Chairmen and the Seniority System. By an unwritten rule, the majority party members who have served longest on particular committees become chairmen. 63

They have formal power over the membership and procedure of their committees and informal influence through their years of experience. They hire staff and can usually kill legislation. Recently, the power of chairmen has been limited, by caucuses led by party leaders. By the Republican takeover in 1994, party leadership had been strengthened at the expense of the committees. Seniority is an important, but unwritten, rule of Congress that has recently been successfully challenged, although it is still generally followed. Advantages of the system are that experienced leadership is assured and rivalries are minimized. Critics, however, feel it is undemocratic and unresponsive to change. They call it the senility system. b. Specialization and Reciprocity. Members of Congress are expected to specialize in the work of their committees, which tends to make them expert in a few areas and unfamiliar with others. Because of this, they rely on each others expertise and follow the lead of other committees on subjects outside their own field. This process has been diluted as party control of the legislative agenda reduced the impact of committee leadership. B. Floor Debate in the House and Senate. Once recommended by a committee, a bill is placed on a calendar for floor debate. In the House, the Speaker controls debate, and each member is limited to five minutes. A simple majority vote of those present is needed for passage. The whole procedure is then repeated in the other house. Voting may follow conservative-liberal lines or regional patterns, but the most common division follows party lines. However, there still are a few mavericks, members who tend to act independently, showing less loyalty to their party in the Congress. They tend to be popular at home, but not in Congress. Party loyalty has increased in both parties, while the parties themselves have become more polarized. Hence, recent congresses have become more partisan, often with a harsh 64

tone to the place (see Congress: Personal, Mean, and Nasty insert). C. Filibuster. A tactic allowed only in the Senate (the House is too large to afford unlimited debate). A Senator can delay or block action by continuously talking. A filibuster can be stopped only by a vote of cloture, in which three-fifths of the Senate votes to stop the debate. Filibusters are most effective late in the session when legislation has piled up. It is frequently used by the minority party to block the majority partys wishes. D. Presidential Veto. The president can stop legislation passed by Congress with a veto. However, Congress can override this veto by a two-thirds vote in each house. The president must act upon a bill within 10 days. If he does not sign the bill, it becomes a law. But if Congress adjourns before the 10 days are up, and the president does not sign the bill, it does not become law. This is called a pocket veto. Note also the fact that Congress retains the advantage of riders, amendments to a bill that a president frequently does not like. Finally, the legislative process must often be carried out twice, since authorization must be followed by appropriations of money. E. The Budget Process. The power of the purse is Congress basic constitutional power. The 1974 Budget Act enabled a congressional alternative to the presidents budget and required rigid deadlines (a completed budget is due by the start of the fiscal year, October 1) for Congress to complete its public spending plan. The Act created House and Senate Budget Committees that guide the Congress in setting total spending, tax, and debt levels. Aiding these committees is the CBO, or Congressional Budget Office. The CBO analyzes the presidents budget proposals and matches Congress spending decisions with the budget targets. Overall spending and tax levels must be set by April 15. . Disagreements over the budget between Clinton and the Republicans led to a government shutdown in late 1995 and 65

early 1996. But by 1998, the budget showed a surplus (due to economic prosperity), with both the president and Congress taking credit. The budget surpluses increased from $69 billion for 1998 to $230 billion for 2000. Debates ensued over what to do with the surplus, with Democrats in the 2000 election calling for the bolstering of social security and an expansion of government programs for education and Medicare. President Bush proposed and was successful in pushing a sizable tax cut of $1.3 trillion. Democrats charged that the surplus would be endangered through this plan. Unfortunately, the Bush tax cut plus the war against terrorism resulted in a growing deficit. The war in Iraq, in 2003, added to that deficit. F. Major Committees in the House 1. Budget. This important committee was established in 1974 by the Budget Act. It allows Congress to propose a comprehensive government budget as a coherent alternative to the presidents budget. Designed to shift power back to Congress, it has not accomplished all that its creators hoped. 2. Rules. All legislation, except money bills, must pass through this committee. Through its power to set the terms of debate, it can delay, amend, or even kill legislation. The committee also acts as a traffic cop for the House Speaker. 3. Ways and Means. Deals with tax and money-raising bills that go to the floor under a closed rulethat is, no amendments can be attached. It has been the key committee on issues such as welfare reform, tax reform, and Social Security reform. This committee was crucial in Bushs 2001 tax cut. 4. Appropriations. This committee (along with Ways and Means) is particularly powerful because of its ability to determine how and where the government will spend its money. It has become a key place for passing pork-barrel bills.

66

5. Judiciary. This committee looks at a broad range of legal matters. It gained unusual public attention during the impeachment hearings of President Clinton. G. Major Committees in the Senate 1. Appropriations. Receives money bills passed by the House and acts as a court of appeals to add money. 2. Finance. Handles tax legislation in the same way as Ways and Means in the House. 3. Foreign Relations. Acts as a watchdog on the presidents dominant position in foreign affairs. Its importance comes from the Senates role in confirming appointments of ambassadors and approving treaties. 4. Rules. The Senates Rules Committee is far less important than its House counterpart. IV. Other Powers of Congress Important non-legislative functions of Congress include oversight of the executive branch, through the right to create and determine the scope of administrative offices, the power to appropriate funds for these agencies, and the authority to investigate any area, activity, or organization that it wishes. Note the role of the GAO, or General Accounting Office. Investigations can act as a check on executive behavior, but the publicity they attract can be dangerous when allowed to get out of hand. Presidential appointments must be approved by a two-thirds majority of the Senate, and confirmation is usually, but not necessarily, assured. A recent example was the House and Senate investigation of President Clintons grant of pardons to financier Marc Rich. There is also the Senate power to approve presidential appointments. Examples include President Bushs controversial appointment of John Ashcroft for Attorney General and Clarence Thomass nomination to the Supreme Court. Impeachment is a slow process and is rarely used, although it was used to impeach President Clinton over the Lewinsky scandal, in 1998. Even though the President was not

67

removed from office, it remains an ultimate power over the executive. V. Wrap-Up Congressional procedures are complex and confusing. Still, Congress has the ability to respond to the peoples demands and needs. If Congress does seem slow and cumbersome, it is because it may be reflecting the diversity of views and strong opinions that exist on policies throughout our vast and heterogeneous nation. Congress was not set up to make the government run more efficiently. It was established to reflect the wishes of the people being governed. VI. Case Study: Campaign Finance Reform Campaign finance reform legislation was signed into law by President Bush in 2002 (even though he termed it "flawed"). The law, dubbed McCain-Feingold in the Senate, banned "soft money" to national political parties and barred corporations, unions, and nonprofit groups from broadcasting "issue ads" that mention a federal candidate within 60 days of a general election or 30 days of a primary (critics claimed this was an abridgement of free speech). In addition, the bill allowed soft money to be given to state/local parties and increased "hard money" individual contributions from $1000 to $2000. Factors contributing to the reform's eventual passage were the 2000 elections, scandals such as Enron, and "legislative pragmatism," which involved compromises and various modifications. However, it seemed that various groups and individuals were trying to weaken McCain-Feingold by finding various loopholes in the law. KEY CHAPTER TERMS AND IDENTIFICATIONS Article I of the Constitution Woodrow Wilson Bicameral (two houses) "national and local" representatives casework for constituents 68

congressional image/scandals the "year of the angry white male" (1994) 108th Congress careerism two-year (House) and six-year (Senate) terms Seventeenth Amendment incumbents term limits, U.S. Term Limits v. Thornton malapportionment; Baker v. Carr gerrymandering packing and cracking "Redistricting following the 2000 census" "population shifts to South and West" Newt Gingrich Speaker of the House (Dennis Hastert) Republican Conference; caucus House Majority Leader (Tom Delay) majority/minority whips House Minority Leader (Nancy Pelosi) Steering Committee/Panel Policy Committee President Pro Tempore GOP or Grand Old Party the frank Senate Majority Leader (Bill Frist) Senate Majority Leader (Tom Daschle) Committee on Committees Republican House Steering Committee Democratic House Steering Panel, Policy Committee Republican Senate Committee on Committees, Policy Committee Standing Committees seniority Conference Committee Select/Special, Joint Committees specialization/reciprocity Calendars; Discharge Calendar North American Free Trade Agreement President George W. Bushs tax cut (2001) Mavericks Senator James Jeffords' defection 69

filibuster/cloture; Rule 22 riders pocket veto, item veto; congressional override 1974 Budget and Impoundment Control Act authorization and appropriation C-SPAN Congressional Budget Office (CBO) budget deficits and surpluses Deficit under George W. Bush standing committees House Committees Budget, Ways & Means, Appropriations, Judiciary, Rules ("closed rule") Senate CommitteesAppropriations, Finance, Foreign Relations, Rules Senator Richard Lugar (R, Indiana) "pork barrel" bills legislative oversight/investigations congressional staffers General Accounting Office (GAO) Impeachment (Clinton-Lewinsky) McCain-Feingold/Shays-Meehan Campaign Finance Reform Legislation Soft Money "elections, scandals, pragmatic legislating" TEACHING SUGGESTIONS 1. If time and circumstances permit, invite a member (or even a former member) of Congress to speak before the class. Interesting insights can be gained from the legislators talk and class questions that would follow. Another option would be to invite a congressional staffer and follow a similar procedure. 2. Discuss in class whether party discipline, especially in the majority party, benefits or harms congressional effectiveness. Is greater party control more democratic? Get opinions from the class. 3. Given the fact that congressional incumbents are re-elected so often, ask the class whether they would support term limits for members of 70

Congress. Should both representatives and senators be limited to, say, 12 years in Congress why or why not? Also, ask members of the class to evaluate the legal reasoning by the Supreme Court when the Court ruled term limits for federal legislators unconstitutional. 4. Why, if Congress is so unpopular, do individual congressmen usually get re-elected? What, if anything, should be done about this? SUGGESTED RESEARCH ASSIGNMENTS 1. Assign a team of students to investigate their senators voting record on five major bills during the current congressional term. Have the team compose a letter to the senator asking him or her the reasoning behind any one or all of those votes. Another approach is to find out which special interests support or oppose a bill. Do those interests donate money to a congressmans campaign? Did the donations influence the vote? 2. Assign short research papers on why filibusters have increased in recent Senates, and how or why they are ended by senators. The students can use the Congressional Digest, Quarterly, or Record in their research. They should try to find examples of filibusters that occurred during the previous six years. 3. Examine the changes in congressional leadership styles since the Republican takeover of the Congress. Did power become centralized or decentralized in the leadership? Were committee chairmen more influential? How was the wielding of power in Congress affected by Dennis Hastert replacing Newt Gingrich as Speaker? 4. Examine the relationship between Congress and President George W. Bush during his first two years in office. How would students characterize the relationship, in terms of policy effectiveness and partisanship? 5. Have students research the implementation of McCain-Feingold. Has the original intent of the law been weakened since its passage, in 71

2002? MULTIPLE-CHOICE QUESTIONS 1. Congress has the power to a. raise taxes. b. determine the nature of the judiciary. c. declare war. d. all of the above 2. The Seventeenth Amendment provided that a. Senators would be elected directly by voters. b. women could vote for congressional candidates. c. Representatives must be at least 25 years of age. d. prohibition be repealed so that members of Congress could now drink openly in the Capitol. 3. Gerrymandering refers to a. drawing oddly shaped congressional districts. b. a political term named after former Massachusetts Governor Elbridge Gerry. c. a and b d. none of the above is correct 4. In recent years, party loyalty in votes in both houses has a. increased. b. decreased. c. increased among Republicans, decreased among Democrats. d. stayed the same

72

5. The whips in Congress a. coordinate party positions. b. act as liaisons between the leadership and the members. c. persuade wavering representatives to vote with the leadership. d. all of the above 6. A caucus is a. a method used to stall debate on the Senate floor. b. needed to override the presidents veto. c. conducted by every newly elected member of Congress. d. a gathering of all members of the same party serving in the House or Senate. 7. The actual drawing of House district boundaries is done by a. the House of Representatives. b. state and federal courts working together in special judicial meetings. c. state legislatures and governors. d. the Supreme Court of the United States. 8. The president pro tempore of the Senate a. is the longest serving member of the Senate from the majority party. b. is the vice president of the U.S. c. is also known as the Speaker of the Senate. d. appoints members of the Rules Committee. 9. Incumbents have a good chance of being re-elected due to a. the usual low voter turnout in congressional races. b. one-party dominance in many districts. c. government-funded trips to the district or state. d. all of the above

73

10. Standing committees in Congress a. have no subcommittees. b. radically change from one Congress to the next. c. are dominated by a small minority who stop legislation by mounting filibusters. d. are the basic working units. 11. The North American Free Trade Agreement is an example of a. voting your party. b. Republicans supporting a Democratic president. c. Democrats supporting a Republican president. d. regional coalitions of Republicans and Democrats voting their consciences. 12. Which of the following is not a characteristic of the Senate in comparison to the House? a. The Senate is more prestigious than the House. b. The Senate is less reliant on staff than the House. c. The Senate is less partisan than the House. d. The Senate is given greater media coverage than the House. 13. In order to override a presidential veto, Congress needs a. three-fifths approval of both chambers. b. a majority of the House and two-thirds of the Senate. c. two-thirds approval of both chambers. d. two-thirds approval only of the House. 14. Under the new campaign finance reform law, soft money may not be given to a. national political parties. b. state political parties. c. both a and b. d. neither a nor b.

74

15. Which career background is most common for legislators in Congress? a. college professors b. bankers c. lawyers d. farmers 16. The Rules Committee can a. delay or stop legislation. b. amend bills or send them back to committee. c. choose one bill to go to the floor, when two committees have bills on the same subject. d. all of the above 17. The Appropriations Committee a. eliminates pork-barrel spending by its 11 subcommittees. b. decides how and when to raise taxes. c. has unlimited spending powers. d. none of the above are considered true 18. The Budget Act of 1974 attempted to give Congress more control over government expenditures by a. enabling Congress to propose an alternative budget to the presidents. b. establishing the Ways and Means Committee in the House. c. revoking the presidents right to submit a budget to Congress. d. establishing a new cabinet post to coordinate the budget. 19. Which of the following presidents was impeached? a. Andrew Johnson b. Richard Nixon c. Bill Clinton d. a and c only

75

20. The seniority system a. favors congressmen from one-party regions. b. is frequently attacked as undemocratic. c. both a and b are true d. both a and b are false 21. Which of the following was not true about the 108th Congress? a. The number of women in the House and Senate reached an all-time high. b. The Democrats controlled the House and the Senate. c. The number of Hispanics in the House increased. d. Nancy Pelosi became the Democrat's House minority leader. 22. Party mavericks are congressmen who a. have increased in number in both parties. b. represent constituencies that are unlike the typical voters of their party. c. both a and b are true d. both a and b are false 23. The Speaker of the House a. is the most powerful in history because of Dennis Hasterts leadership. b. is also chairman of the Rules Committee. c. exercised almost dictatorial powers at the turn of the twentieth century. d. is the longest serving member of Congress. 24. Because all money-raising bills must begin in the House of Representatives, any tax legislation must be sent first to the a. Rules Committee. b. Ways and Means Committee. c. Appropriations Committee. d. Energy and Commerce Committee. 25. Congressional staff can do the following: a. organize hearings b. promote legislation 76

c. initiate proposals d. all of the above 26. Conference committees a. are temporary bodies with members from the House and Senate. b. are comprised of all Republican members. c. meet to override a presidential veto. d. none of the above 27. The Senate Special Committee on Whitewater illustrates a. Congress power to withhold appropriations. b. actions by joint committees. c. Congresss power to investigate. d. the power of judicial review. 28. Rule 22 of the Senate allows a member to a. serve as president pro tem. b. talk as long as they wish about any subject. c. serve on both the Finance and Appropriations Committees. d. filibuster until three-fifths of the Senate votes to end debate. 29. The number of total congressional employees is closest to what number? a. 35,000. b. 5,000. c. 25,000 d. 15,000.

77

30. Under campaign finance reform, hard money contribution limits by individuals were changed from $1000 to a. $2000. b. $1500 c. $3000 d. $2500 TRUE-FALSE QUESTIONS 1. C-SPAN covers both houses of Congress. 2. A gerrymander can only occur in the Senate. 3. The 1974 Budget Act created a new cabinet position to help pass the budget. 4. There is no way to override a presidential veto. 5. Until 1994, the Democrats had been the majority party in Congress, retaining unbroken control of both the House and Senate for the past forty years. 6. The Rules Committee disbands after reconciling two versions of a bill. 7. The vast majority of cloture votes in the Senate are usually successful. 8. The filibuster can only occur in the Senate. 9. Over 90% of incumbents in Congress running for re-election win. 10. Since the position is mentioned in the Constitution, the president pro tem of the Senate is among the most influential in Congress. ESSAY QUESTIONS

78

1. Congress is often charged as being unrepresentative and ineffective. How would you defend Congress against this criticism? Use the profile and performance of the 108th Congress in your answer. 2. Discuss how the House of Representatives differs from the Senate in structure and operating procedures. How would one expect the behavior of the two houses to differ? Give examples. 3. It is often said that the Senate is the most exclusive millionaires club in the world. Research the backgrounds of a few Senators. How did wealth or family name help/hinder their election? Is the Senate still a millionaires club? Does it matter? 4. What factors led to the successful passage of campaign finance reform legislation? Will this legislation change the nature of American politics--why or why not? 5. Explain why incumbents have distinct advantages over challengers. 6. Explain the relationship between a new national census and the impact that census has upon reapportionment of Congressional seats. 7. List the reasons why so many proposed bills never get through both houses of Congress. 8. How do the rules of specialization and reciprocity support the power of committees in Congress? 9. Explain the specific roles of the CBO and GAO. 10. Why does the author assert that Congress was not set up to make government run more efficiently? Chapter 4 Multiple Choice 1. D 2. A 6. D 7. C 3. C 8. A 79 4. A 9. D 5. D 10. D

11. 16. 21. 26.

B D B A

12. 17. 22. 27.

B D B C

13. C 18. A 23. C 28. D 3. F 8. T

14. 19. 24. 29.

A D B A

15. 20. 25. 30.

C C D A

True-False 1. T 6. F

2. F 7. F

4. F 9. T

5. F 10. F

80

CHAPTER FIVE The Judicial Branch: The Supreme Court and the Federal Court System

CHAPTER OVERVIEW This chapter focuses on the Supreme Court, with less emphasis on the other federal courts, i.e., U.S. District Courts and Courts of Appeals, their structure, and jurisdiction. The Supreme Courts historical evolution is covered, along with the Courts internal and external limits. An important theme is the Supreme Courts role as a political player. The two competing approaches to judicial authority--activism and restraint-are also surveyed. The chapter ends with a case study on racial segregation, including the eventual judicial shift from Plessy v. Ferguson to Brown v. Board of Education. LEARNING OBJECTIVES After reading chapter 5, the student should be able to: 1. Diagram the federal court structure, noting the respective patterns of appellate and original jurisdiction. 2. Explain the operation of the U.S. Supreme Court and such related terms as "writ of certiorari" and "judicial review." 82

3. Outline the important stages of the Supreme Courts history, noting key cases and their significance in each historical era. 4. Summarize the important internal and external limits operating upon the Court. 5. Explain how and why the Court is such an important political player in the U.S. political system. 6. Distinguish between judicial activism and judicial restraint. 7. Understand the evolution of the Court and the political systems treatment of racial segregation. 8. Explain the significance of the Courts decision in Bush v. Gore vis-vis the 2000 presidential election. OUTLINE I. The Federal Court System A. U.S. District Courts. This is the base of the federal court system, the courts of original jurisdiction. Although there are 94 of them, the large volume of cases (almost 260,000) they handle has led to long delays in administering justice. Cases handled by these courts include civil rights issues and counterfeiting. Courts of Appeals. There are thirteen, which handle appeals from the district courts and from important regulatory commissions. Very few cases go beyond this level to the Supreme Court. One hundred seventy-nine circuit court judges handle almost 55,000 cases annually.

B.

83

C.

Special Federal Courts. These courts were created by Congress to handle certain cases. The Claims Court deals with peoples claims against government seizure of property, while the Court of Military Appeals, composed of three civilians, is the final judge of court-martial convictions. Federal Judges. Nominated by the president and confirmed by the Senate. To preserve their impartiality, judges hold office for life and can only be removed by impeachment. They cannot have their salaries reduced. Their appointments, however, are usually on a party basis. For example, 93 percent of LBJs appointments to federal judgeships were Democrats, whereas 93 percent of Richard Nixons were Republicans. Both Bush and Reagan selected many conservative judges, while Clinton made more liberal appointments, including two to the Supreme Court. Clinton appointed 373 federal judges by the end of his second term. Clinton appointed Ruth Bader Ginsburg, in 1993, and Stephen G. Breyer, in 1994, to the Supreme Court. Both have acted as a brake on the Courts conservative tendencies. Ginsburg had a particularly vigorous dissent in the Courts 2000 decision involving the stopping of the voting recount in Floridas presidential election. A majority of the Supreme Court has been appointed by Reagan and Bush. Reagan appointed the first woman to the Supreme Court, Sandra Day OConnor, in 1981. Reagan also selected Anthony Kennedy, and Bush selected David Souter (conservatives later considered this a mistake). In addition, Senate confirmation hearings have become testier (Bork, Thomas), and this has continued during the Bush Administration, given the pattern of divided government. In 2002, President Bush accused Democrats in the Senate of not filling 88 vacancies for the federal courts.

D.

84

E.

Jurisdiction. Refers to the matters over which a court may exercise its authority. The federal courts have jurisdiction over all disputes relating to the Constitution and treaties of the United States (jurisdiction assigned by the subject), as well as over all cases involving the United States as a party, two states or two citizens of separate states, ambassadors, or a citizen of the United States and a foreign citizen (jurisdiction assigned because of the parties). The courts have either exclusive jurisdiction or concurrent jurisdiction, which they share with state courts.

II.

The United States Supreme Court The membership includes one chief justice and eight associate justices (Congress can set by law the number of justices on the Supreme Court). A very small number of the total cases handled by the federal court system ever reach the Supreme Court. Of more than 10 million cases tried in American courts, only 7,400 petitions for review make it to the Supreme Court. The Court agrees to consider only about 140 cases in a term. Most cases appear only in the form of petitions for a writ of certiorari, which is an order from a higher to a lower court, requesting the records of a case. The Court denies 85-90 percent of these petitions. The Court has both original and appellate jurisdiction. A. The Final Authority? The Supreme Court is said to be the final authority, but a ruling may be reversed by a later Court ruling or by an amendment to the Constitution. (An 1895 Court decision striking down the federal income tax was reversed by the Sixteenth Amendment, which legalized that tax.) The Court also shares its power to interpret the constitution with the executive and legislative branches. Public opinion may also not be receptive to a ruling (school prayer).

85

B.

Early Years of the Court/Judicial Review and National Supremacy. In its first three years, no cases at all were brought to the Supreme Court. But two early decisions greatly increased the power of the Court. In Marbury v. Madison (1803) the Court invalidated an act of Congress for the first time, establishing the principle of judicial review; McCulloch v. Maryland (1819) held that the federal government is supreme within its sphere of powers. John Marshall, perhaps the Courts greatest leader, was Chief Justice for both cases. In 1857, the Supreme Court invalidated the 1820 Missouri Compromise by its ruling in the Dred Scott case. The ruling represented a major expansion of the courts judicial powers. The Court After the Civil War. Since the Civil War, the Court became both more active and more conservative in its responses to the growth of government regulation of the economy. Many argue that the Court simply protected property rights of the rich and ignored public demands for government regulation. It continued to resist the expansion of state and federal regulatory power until 1937, when it was forced to leave the arena of economic policy-making, after a confrontation with Roosevelt. FDR also tried to pack the court, albeit unsuccessfully. The Modern Courts. The Supreme Court today shows three major trends: (1) it has struck down much less legislation than did earlier Courts; (2) it has avoided protecting private property rights against government regulation; and (3) it has shown increased concern for the rights of individuals against the growing power of government. The Court has taken a leading role in eliminating racial discrimination, in the reapportionment of election districts (Baker v. Carr), and in protecting the rights of criminal defendants (Miranda v. Arizona). The Burger Court, while less active than the Warren Court before it, made major decisions by legalizing abortion, allowing publication of the Pentagon Papers, allowing local communities to define 86

C.

D.

obscenity, and restricting affirmative action programs. The Burger Court did leave the Miranda decision in place. The Rehnquist Court (1986- ). The Rehnquist Court has embraced judicial activism in support of federalism. It has invalidated twice as many federal laws as the activist Warren Court. It has struck down both conservative positions (stopping the execution of mentally retarded offenders) and liberal laws (allowing random drug tests of high school students; the Bush v. Gore decision). However, during the 1995 term, the Court did move in a clearly conservative decision in the areas of race and religion (Rosenberger v. Rector and Visitors of University of Virginia), approving government funding for a religious activity. The Court also allowed government vouchers in its 2002 Zelman v. Simmons-Harris ruling. In the area of term limits, the Court ruled, in 1998, that the presidents line-item veto authority was not constitutional (Clinton v. City of New York). Other areas affected included limiting the power of Congress to regulate interstate commerce (United States v. Lopez), throwing out a congressional law that allowed victims of violence to sue in federal court against their assailants (U.S. v. Morrison), and ruling that federal agencies may not hear complaints by private parties against states (Federal Maritime Commission v. South Carolina Ports Authority). In short, the focus of the Court seems to be on the federal governmentincluding Congress which it views as too big, too powerful, and often, too incompetent. Whether this perspective will continue will depend on what type of Justices will replace those nearing retirement (at least 3 justices are over 70 years of age). III. Strengths and Weaknesses of the Supreme Court A. Internal Limits on the Court. The internal limits include the need for an actual case, the Courts refusal to resolve political 87

questions, the avoidance of constitutional issues whenever possible, and the tendency to follow precedent (stare decisis). Note that regarding a political question, the Court is flexible, as exemplified by Baker v. Carr and Bush v. Gore. B. External Limits. These involve congressional powers to limit the Courts jurisdiction and to approve Court nominees. With no army or bureaucracy to enforce its decisions, the Courts authority is dependent upon the other branches of the government and upon all the political players accepting their opinions. Strengths of the Court. Its enormous prestige, the fragmented nature of the American constitutional structure, and the legal profession (role of the ABA), which acts in large part as the courts constituency, all add to the Courts strengths. The courts prestige is unquestionable. These supports and the Courts selflimitations have preserved the unique tradition of judicial review and the central importance of the Supreme Court in the political game.

C.

IV. The Court as a Political Player A. Introduction. The Court is a political as well as a legal institution; it makes decisions that set public policy. Its authority remains dependent on the acceptance of its decisions by other branches of the government and by the public. The Court cannot ignore the reactions to its decisions in Congress or in the nation. Judicial Restraint Versus Judicial Activism. These are the two competing approaches to the use of judicial authority. Judicial restraint calls for the Court to play a passive role, allowing the other branches of government to lead the way in shaping policy. Judicial activism sees the Court as an active partner with the other branches in resolving problems facing the country. The 88

B.

Warren Court exemplified judicial activism. Judicial activism and restraint should not be confused with liberal vs. conservative, as recent examples of judicial activism include the Rehnquist Court decisions limiting federal authority over the states and halting the vote count in the 2000 Florida election, both of which were seen as conservative reversals of liberal policies. V. Case Study: Separate but Equal? The Courts role as a political player is revealed in the changing political forces surrounding the issue of racial segregation. After the Civil War, attempts were made by radical Republicans, who controlled Congress, to guarantee full civil rights to blacks. However, both the increasing power of the white South and Supreme Court rulings negated these civil rights laws. In the Plessy v. Ferguson decision of 1896, the Court approved the doctrine of separate but equal, thereby sanctioning legal segregation of the races through state Jim Crow laws. Although the court increasingly undercut the doctrine of segregation, starting in the 1930s, it did not overrule Plessy until 1954, in Brown v. Board of Education. After this decision, the Court moved to encourage integration in a number of areas of American life and to support the civil rights movement. The Courts tardiness in overruling Plessy, as well as its present reluctance to vigorously combat discrimination, reflects the larger political climate and the political limits in which the Court operates. KEY CHAPTER TERMS AND IDENTIFICATIONS Article III, Section 1 of the Constitution State courts U.S. District Courts original and appellate jurisdiction (appeals) U.S. Courts of Appeals or Circuit Courts of Appeal 89

special federal courts U.S. Claims Court U.S. Court of Military Appeals Robert H. Bork Alcee Hastings A "vacancy crisis" in the federal courts Anita Hill and Clarence Thomas Chief Justice Earl Warren Sandra Day OConnor, Ruth Bader Ginsburg Stephen G. Breyer Senatorial courtesy exclusive and concurrent jurisdiction subject of the case; parties to the case Supreme Court of the United States Memorandum orders writ of certiorari Dred Scott decision (1857) and Missouri Compromise (1820) Marbury v. Madison (1803) and judicial review McCulloch v. Maryland (1819) and national supremacy Implied powers Dred Scott v. Sandford (1857) and property rights (slaves) Chief Justice John Marshall States rights vs. federal powers FDRs court-packing scheme and New Deal reapportionment Baker v. Carr (1962)--"one man, one vote" Miranda v. Arizona (1966) Pentagon Papers political questions Warren Court Burger Court Rehnquist Court Bush v. Gore (2000) Rosenberger v. Recto and Visitors of University of Virginia (1995) Zelman v. Simmons-Harris (2002)--allowing government vouchers 90

U.S. Term Limits Inc. v. Thornton United States v. Lopez (1995) Clinton v. City of New York (1997) Federal Maritime Commission v. South Carolina Port Authority (2002) Dickerson v. United States (2000) U.S. v. Nixon (1974) Political Questions and constitutional issues precedent or stare decisis Judicial Restraint vs. Judicial Activism Courts prestige; fragmented nature of American governmental system ABA Thirteenth, Fourteenth, and Fifteenth Amendments Jim Crow laws, Sundown Ordinances Plessy v. Ferguson (1896) Brown v. Board of Education (1954) Civil Rights Act of 1964 Affirmative action TEACHING SUGGESTIONS 1. Review the backgrounds and judicial philosophies of the nine current justices of the Supreme Court. Ask members of the class to consider which justices comprise the liberal faction and which justices belong to the conservative coalition. Then, examine a couple of current Supreme Court rulings to demonstrate how the Courts internal fragmentation, especially the recent 5-4 divisions, affected these rulings. 2. Arrange a debate between two student teams regarding the merits and demerits of judicial activism versus judicial restraint. Make sure both liberals and conservatives are represented on both sides. The students can prepare short briefing papers on the two judicial philosophies, which may be distributed to other members of the class prior to the 91

debate. The teams may wish to use Bush v. Gore as part of their presentations. SUGGESTED RESEARCH ASSIGNMENTS 1. Ask a panel of students to research the kinds of pressures placed upon Supreme Court nominees when they undergo Senate confirmation. The students can research those rejected examples, such as Bork and Carswell, controversial nominees such as Clarence Thomas, and successful nominees such as Sandra Day OConnor or Clintons 1994 choice, Stephen Breyer. 2. Divide the class into three sub-groups. Let one group prepare an oral report on the current Courts attitude toward federalism. Another group can report on the latest rulings affecting civil rights, while a third group can handle cases affecting police powers. MULTIPLE-CHOICE QUESTIONS 1. Which federal court was established by the Constitution? a. the Courts of Appeals b. the United States District Courts c. the Supreme Court d. all of the above There are _____ United States district courts. a. 94 b. up to 15 c. 11 (plus one in Washington, D.C.) d. 179

2.

92

3.

Judicial review is a. the power to declare acts of any state or local government unconstitutional. b. the power to declare laws of any state or local government unconstitutional. c. the power to strike down acts of any branch of the federal government. d. all of the above Which case established the principle of judicial review? a. Roe v. Wade b. McCulloch v. Maryland c. Marbury v. Madison d. Brown v. Board of Education Which of the following insulates judges from political pressures? a. they are not allowed to vote in federal elections b. appointment for life c. their salaries cannot be reduced d. both b and c are correct The United States Claims Court a. was created by the military to court-martial soldiers. b. addresses civil rights cases only. c. is known for showing judicial restraint. d. was created by Congress to deal with peoples claims against government seizure of property. The Warren Courts Baker v. Carr decision dealt with a. apportionment. b. abortion rights. c. segregation. d. affirmative action.

4.

5.

6.

7.

93

8.

The first woman appointed to the Supreme Court was a. Ruth Bader Ginsburg. b. Anita Hill. c. Sandra Day OConnor. d. Madeline Albright. Congress has the right to impose which of the following limits on the Court? a. It can set when and how often the court will meet b. It can establish the number of justices c. It can limit the Courts jurisdiction d. all of the above

9.

10. Senatorial courtesy a. gives a senator a role in appointing judges. b. means that questioning judicial nominees follows committee seniority. c. played a role in confirming judges in the nineteenth century, but is no longer practiced. d. was a judicial theory expounded on by Warren Burger. 11. Judicial activism a. is closely identified with the Warren court. b. views the court as an active participant in shaping government policy. c. may result in conservative as well as liberal positions. d. all of the above 12. The case of Plessy v. Ferguson upheld a. racial segregation. b. affirmative action. c. habeas corpus. d. none of the above

94

13. Which Chief Justice is considered the most influential in U.S. history? a. William Rehnquist b. Roger B. Taney c. John Marshall d. John Jay 14. Dred Scott v. Sandford (1857) a. invalidated the Missouri Compromise. b. upheld the Wilmot Proviso. c. declared the one man, one vote rule. d. challenged the right to own slaves. 15. Miranda v. Arizona (1966) a. is an example of a voting rights case. b. is an example of a racial discrimination case. c. is an example of a criminal defendant case. d. is an example of a states right case. 16. Who appointed the second female justice? a. George Bush b. Ronald Reagan c. Bill Clinton d. Jimmy Carter 17. Considered the least dangerous branch of government, the Supreme Court derives its influence from a. the fragmented nature of the American constitutional structure. b. the legal profession. c. being able to use the police to enforce the law. d. a and b only

95

18. Between 1900 and the mid-1930s, the Supreme Court exercised judicial review primarily in what kinds of cases? a. economic regulatory cases b. civil liberties c. separation of powers d. racial discrimination 19. Judicial activism a. was the philosophy that guided Oliver Wendell Holmes, Jr. b. was rejected by the Warren Court. c. is the idea that the Courts should play a central role in shaping government policy. d. all of the above 20. In United States v. Lopez, the Court a. legalized betting on horse races. b. limited Congress ability to regulate interstate commerce. c. established judicial review. d. limited states' rights to set term limits. 21. If an individual took a civil rights case to a U.S. District Court and lost, he or she could appeal the case to the next level in the federal court structure, which would be the a. U.S. Supreme Court. b. U.S. Court of Appeals. c. U.S. Court of Military Appeals. d. U.S. Claims Court. 22. Robert H. Bork is significant because a. he was the first African American nominated to the Supreme Court. b. he was a longtime advocate of judicial activism. c. he was defeated in a bitter battle over the ideological composition of the Court. d. he was the last non-lawyer to serve on the Supreme Court. 96

23. Of the more than 10 million cases tried every year in American courts, how many petitions for review make it to the Supreme Court? a. about 500,000 b. about 7,400 c. about 1,000 d. about 25 24. Court-packing refers to a. appointing only judges of your party to the bench. b. intimidating judges by filling the court with rowdy observers. c. the occasion when the judges rule on more cases than the norm. d. none of the above 25. In Plessy v. Ferguson, the Supreme Court ruled that a. segregation was related to the superiority of the white race. b. slaves were property and therefore had no rights under the Constitution. c. segregation was in and of itself contrary to the Fourteenth Amendment. d. none of the above 26. The Supreme Court case of Dickerson v. United States (2000) dealt with a. the Miranda ruling b. the Pentagon Papers c. Roe v. Wade d. United States v. Nixon 27. President Clinton appointed the following justices a. Thomas and Souter b. Ginsburg and Thomas c. Ginsburg and Scalia d. Ginsburg and Breyer

97

28.

The Rehnquist Court has recently limited a. federal powers to regulate abortion. b. federal powers to regulate interstate commerce. c. federal powers to set term limits. d. anyones right to burn flags. The doctrine of "political questions" a. was established by William Howard Taft. b. cannot be reversed once it has been applied to a case. c. allowed the courts to avoid ruling on President Bushs use of force against Iraq. d. none of the above The United States has approximately how many lawyers? a. 124,000 b. 780,000 c. 27,700 d. 124,000

29.

30.

TRUE-FALSE QUESTIONS 1. 2. 3. 4. The Sixteenth Amendment overturned a Supreme Court holding that the property tax was unconstitutional. Under the doctrine of precedent, the Supreme Court cannot overturn earlier decisions. Though there are up to 15 judges on an appeals court, only three usually hear each case. The doctrine of national supremacy is found in the Preamble to the Constitution.

98

5. 6.

A writ of certiorari is an order to any lower court to send the entire record of a case to a higher court for review. There have always been nine judges on the Supreme Court.

7. Partisanship never plays a role in appointing judges. 8. The most active Supreme Court terms were the first years of the Republic, because the Court had to rule on a large number of separation of powers cases. Political conservatives are in favor of judicial restraint.

9.

10. The Supreme Court is a political institution that sets national policy by interpreting the law. ESSAY QUESTIONS 1. In what ways is the federal court system responsive to political issues? In what ways is it shielded from political pressures? 2. Discuss the relationship between judicial activism and judicial restraint, and liberalism and conservatism. How do the Warren and Rehnquist Courts conform to these terms? 3. Discuss the role that partisanship plays in appointing and confirming judges. 4. The author claims that the Supreme Court is clearly the weakest of the three branches. Has the court gotten weaker or stronger in recent years? Explain.

99

5. Is there evidence that the overturning of the separate but equal doctrine, in Brown v. Board of Education, was not a sudden reversal? Discuss, citing historical details 6. What was the historic significance of Bush v. Gore? Why did Justice Stevens argue that the decision harmed the nations confidence in judges? 7. Which Supreme Court Justices have disappointed those presidents who originally appointed them? Cite some specific examples. 8. From a historical perspective, why does the author assert that one of ironies of American democracy is that the least democratic branch is also the most popular? 9. What was the legal significance of the following recent rulings by the Rehnquist Court(a) Clinton v. City of New York; (b) U.S. v. Morrison; (c) Federal Maritime Commission v. South Carolina Ports Authority? 10. Why and how does the Supreme Court avoid political questions? Chapter 5 Multiple Choice 1. C 2. A 6. D 7. A 11. D 12. A 16. C 17. D 21. B 22. C 26. A 27. D True-False 1. F 6. F 2. F 7. F 3. D 8. C 13. C 18. A 23. B 28. B 3. T 8. F 100 4. C 9. D 14. A 19. C 24. D 29. C 4. F 9. T 5. D 10. A 15. C 20. B 25. D 30. B 5. T 10. T

CHAPTER SIX Civil Rights and Liberties: Protecting the Players

CHAPTER OVERVIEW This chapter distinguishes between civil rights and civil liberties, with a number of real-life examples. The debate between those who advocate a partial incorporation of the Bill of Rights into the Fourteenth amendment, as it applies to state governments, and those who suggest a complete incorporation is also covered. The four most important civil liberties-speech, religion, privacy, and due process-- are discussed fully. The chapter then proceeds to the area of civil rights. Specific coverage is given to race and sex, as suspect classifications, as well as to the actors in civil liberties and rights--judges (activism vs. restraint), legal defense funds, private attorneys general, state agencies, and the public. The chapter concludes with a case study on protecting liberties during the U.S. war against terrorism. LEARNING OBJECTIVES After reading chapter 6, the student should be able to: 1. Distinguish between the meanings and legal implications of civil rights and civil liberties. 2. Explain the doctrine of incorporation and the debate that surrounds it. 101

3. Apply recent court decisions to the four most important civil liberties, explaining how those liberties have been modified. 4. Define such important terms as suspect classifications, equity, judicial activism/restraint, private attorneys general, landmark decisions, injunctions, exclusionary rule, and affirmative action. 5. Summarize the key points from the chapters case study on "Fighting Terror. Protecting Liberties." OUTLINE I. Civil Rights and Liberties in the Game Analogy. These important rules that protect the players govern both the legitimate means and ends of the political process. They tell us how to play the game, as well as why we ought to play (e.g., to protect freedom of speech and religion). They are based on two principles: that the government must not violate the rights of its citizens, and that government has an obligation to provide equal protection under the law for all groups in society. This chapter focuses on how the courts and other players protect civil rights and liberties. What are Civil Liberties and Rights? A. Civil Liberties. Defined as a set of protections against government restrictions on the rights of speech, petition, assembly, and the press. These rights protect people when they participate in the democratic political system. American government is one of laws, guaranteeing each citizen due process proceedings to ensure individual liberties. Protection of civil liberties is central to such issues as the right to criticize and oppose current policies, such as government censorship, and the right to organize for elections. 102

II.

B.

Civil Rights. Defined as a set of protections (granted in the Fourteenth Amendment) against discrimination on the basis of race, religion, ethnicity, or gender. The Constitution recognizes that all citizens must be treated equally under the laws and, therefore, forbids discriminatory treatment of minorities or other groups. Conversely, no racial, religious, or ethnic group can claim privileged treatment. Everyone is affected by civil rights issues.

III.

Expanding the Bill of Rights The original Bill of Rights applied only to the federal government. It has been expanded, through the use of the Fourteenth Amendment, to apply to state governments and individuals, and extended to cover new issues, such as due process rights in private institutions like universities. A. Uses of the Fourteenth Amendment. The due process and equal protection clauses of this amendment, ratified in 1868 after the Civil War, have been used by the federal courts to expand the application of the Bill of Rights to state governments, as well as the federal government. The equal protection clause has been applied to racial and sex discrimination in areas involving state interests or laws. It has also prevented discrimination by private individuals, when their actions are aided by the state, furthers state activity, or involves a fundamental state interest. The vague phraseology of the due process clause has led to a debate over the extent to which the Fourteenth Amendment incorporated the Bill of Rights. B. Partial Incorporationists versus Complete Incorporationists. These are two competing judicial interpretations of the Fourteenth Amendment. Partial incorporationists argue that only some parts of the Bill of Rights should be included in the meaning of due process. They point to procedures 103

guaranteeing fair trials and the First Amendment freedoms. In short, preferred freedomsliberties necessary for a democracy to functionshould be incorporated. The courts must decide applicability in each case. Complete incorporationists argue that every provision of the Bill of Rights is covered by the Fourteenth Amendment and must be recognized by the state courts. While the Supreme Court has never adopted full incorporation, the cumulative effect of federal court decisions has been to incorporate almost all of the Bill of Rights into the Fourteenth Amendment. IV. Civil Liberties: Protecting People From Government Civil liberties provide protection for players in the political game against abuses of government authority, while making sure electoral losers have a fair chance to get back into power. These rules are essential to allowing democracy to work. They enable people to keep informed and to communicate with each other, and with the government, without fear. Indeed, Supreme Court Justice Oliver Wendell Holmes wrote that a democratic society needs competition among ideas as much as it needs economic competition. Fundamental to Holmess thinking was that good ideas will drive bad ideas out of the market, with the public rejecting the false over the true. However, propaganda may sometimes overwhelm reason. A closer look at four important liberties (freedom of speech, freedom of religion, right to privacy, and due process) demonstrates their importance to the democratic processes of open discussion, opposition, and reform. A. Freedom of Speech. This guarantee has been expanded to state governments under the Fourteenth Amendment, and the definition has been broadened to include such things as membership in organizations, public demonstrations, leafleting, symbolic speech (speech plusincluding burning flags or wearing buttons with a message), and, with some debate, speech 104

in cyberspace. The Supreme Court upheld congressional laws that make it a crime to conspire to overthrow the government by force, but belief in revolution is not a crime as long as conspiracy or an actual act cannot be proven. However, the First Amendment does not protect speech that motivates listeners to illegal conduct or, apparently, politically incorrect acts on some college campuses. The First Amendment also protects you by prohibiting the government from forcing you to say anything you dont believe (i.e., Jehovahs Witnesses and Pledge of Allegiance case). The First Amendment does not protect speech that leads to illegal conduct (yelling fire in a crowded theater when there is no fire), libel or slander, making or selling child pornography, or fighting words. However, free speech online, even if pornographic, has been protected by the Supreme Court in the Reno et al. V. ACLU et al. Ruling (1997). See Freedom on the Internet text insert. B. Freedom of Religion. Although there has never been a complete separation of church and state in America, the increasing political power of Christian fundamentalists has led them to attempt to redefine church-state relations. A constitutional amendment to allow school prayer is an example of the freedom of religion controversy, and recent government guidelines have attempted to navigate this controversy. Most Americans favor prayer in school, viewing it as an issue of the free exercise of religion. However, most Americans also believe that government must not favor one religion over another. Yet, there has never been complete church-state separation. Note President George W. Bushs plan to give government funds to faith-based groups for their charitable work. Also, note the 2002 Supreme Court ruling that educational vouchers were neutral in respect to religion, even though those government105

funded vouchers could be used by parents to send their children to religious schools. See Dos and Donts on Religion in the Public Schools text insert. C. Rights of Privacy. Protection from government intrusion into the private lives and beliefs of citizens is guaranteed by the First and Ninth Amendments. Although not explicitly mentioned in the Constitution, the courts have created a zone of privacy, guarding individuals from government intrusion. Issues of sexual conduct between adults (the state cannot prevent couples from using contraceptive devices) and abortion (states cannot forbid abortions in the first three months of pregnancy) are recent examples of the right to privacy. But the courts have also placed limits on some of these freedoms. For example, homosexual marriages need not be recognized by the states and states are not required to fund abortions for those individuals who cannot pay for them. Finally, in Romer v. Evans (1996), the Supreme Court ruled that Colorado could not single out gays and prohibit local laws designed to end discrimination against them. Due Process Rights. These rights involve fundamental procedural fairness and impartial rulings by government officials, especially in criminal procedures. For example, these procedural guarantees include the presentation of charges, the right to a lawyer, time to prepare a defense, a speedy and fair trial by an impartial judge and jury of peers, and the right to appeal. These rights have been extended to state criminal trials and to other settings, such as educational and social service hearings. These rights were granted in federal criminal trials under the Fifth and Sixth Amendments.

D.

V.

Civil Rights: Protecting People From People

106

The government can assume a positive role in protecting the rights of minorities of race, religion, sex, or national origin. Minorities may seek redress for discrimination through the political or judicial system. Of course, civil rights issues are not always clear cut. Minorities that achieve the status of protected class are eligible for inclusion in affirmative action programs. Ironically, minorities seeking protection under affirmative action programs increasingly come into conflict with one another, rather than with the majority. Nevertheless, these programs have also fueled white resentment against reverse discrimination. A. Which People Need Protection? Suspect Classifications. A limit placed by the courts upon the governments ability to classify and legislate for certain groups. With respect to any economic issue, the laws passed by Congress or state legislatures are routinely approved by the judiciary, under the doctrine of presumptive legislative rationality. On the other hand, if lawmakers apply racial or religious classifications, the courts subject these to close scrutiny, because these are suspect classifications. The burden of proof is on the government to prove that laws for a suspect category (i.e., a racial group) are necessary. Racial classifications are almost always considered suspect and are therefore struck down. Courts usually strike down most gender classifications, but a few are upheld, such as women not having to register for the draft.

B. Race as a Suspect Classification. In 1896, the Supreme Court in Plessy v. Ferguson supported state actions that segregated the races. But over a half-century later, in the landmark case of Brown v. Board of Education, the Court reversed itself and established race as a suspect classification. The courts then struck down all laws based on race. This has been modified to allow government to make laws that will serve to eliminate prior inequities caused by state-sponsored segregation. But the Rehnquist Courts 1989 decision in City of Richmond v. Croson 107

and Adarand v. Pena (1995) seemed to throw all racial classifications into question. While not declaring all affirmative action programs unconstitutional (the 1978 Bakke case had seen the principle of affirmative action upheld), the Court made clear that there had to be a compelling interest to relieve a specific case of discrimination. Most existing programs were not expected to survive the Courts strict scrutiny. Other branches of government have responded to the Courts actions. In California, voters passed Proposition 227, which banned the use of race and sex in college admissions and public employment. The argument over affirmative action continues, as noted in the text by the Coleman-Sowell debate. C. Is Sex Suspect? Laws based on sex were supported in the past, because they were thought to protect women. These laws were known as protective legislation. The womens movement has objected to these laws as paternalistic, but sex remains an uncertain suspect category. The federal courts do retain part of the protective movement (i.e., sexual harassment laws). The Supreme Court has struck down many laws based on sex, although the rule of intention favors the government because victims of sex discrimination must prove that officials intended discrimination. It is likely that more sex-based classifications will fall as a result of present standards applied by the courts.

VI. Actors in Civil Liberties and Rights The government, as well as voluntary organizations, are players in the political game, attempting to strike a balance between competing claims. A. Judges. Judges play a leading role in developing civil rights and liberties. Two approaches to the use of law shape society. Activist judges, along with those called civil libertarians, may support civil liberties and rights by backing class action suits 108

that have wider implications, or by applying the concept of equity. Equitable remedies allow the courts to prevent future discrimination by shaping remedies not covered by existing law, such as requiring school districts to develop plans overcoming racial imbalances. Non-activist judges are unsympathetic to class actions and tend to stress existing policies developed by Congress and the president. They tend to follow past decisions rather than expand the scope of constitutional protections. They see elected officials as being more accountable to the people. B. The Justice Department. Historically, the Department of Justice has played a key role in protecting civil rights and liberties. But its role has varied by administration. The Department in the Reagan and Bush administrations opposed many civil rights measures, but the Clinton administration has been generally supportive of civil rights groups and causes in a quiet way. George W. Bush, elected with little black support, has pursued a moderately conservative course toward civil rights, although opposing affirmative action. Bushs Attorney General John Ashcroft is likely to be less active in civil rights enforcement. Private Attorneys General. These are privately supported groups that sponsor cases against the government or other groups. They are funded in part by foundations and wealthy donors, and in part by dues-paying members. The largest of these, the American Civil Liberties Union (ACLU), handles more than 6,000 cases a year defending the rights of such groups as women, minorities, students, and minors. Legal defense funds have also been formed by minority group associations, such as the NAACP Legal Defense and Educational Fund. These focus their efforts against discrimination in housing, schools, employment, voting rights, and the legal system. Legal Strategies. These organizations use a variety of legal tactics. They search for patterns of discrimination, they write 109

C.

D.

articles in law journals, they offer services to individuals whose rights may have been violated. Organizations such as the ACLU concentrate on offering their services to individuals unable to pursue their cases alone, in the hopes of finding a test case (i.e., one that has a good chance of setting a new precedent). These legal groups hope that their case will be argued in the Supreme Court, where it will become a landmark decision, creating a new legal standard that affects later decisions in the same area. E. Obeying the Courts. In addition to declaring state laws unconstitutional, the courts can issue injunctions and court orders to ensure that rights are protected. Federal orders must be obeyed by state officials and can be enforced by citations, fines, jail sentences, and the use of federal marshals or, as both Eisenhower and Kennedy did, troops. Compliance often requires pressure from federal authorities, as was the case with police departments following the warning guidelines of Miranda. The police were further pressured by the courts applying the exclusionary rule. Public Opinion and Civil Liberties. Judicial decisions are often not supported by public opinion. A majority of the public does not believe that criminals should be released on legal technicalities. The judiciary is not directly accountable to the public, and debate over its pronouncements is a sign of health in the system, although dissent may interfere with compliance.

F.

VII. Case Study: Fighting Terror and Protecting Liberties After 9/11, a potential conflict emerged between security against terrorist attacks and the Constitution's civil liberties. Over 1200 people suspected of terrorist links were locked up without judicial review, counsel, or open hearings. Historically, the executive branch has abridged liberties (Lincoln, Wilson, FDR), and the courts, along with public opinion, have supported those actions. The Bush 110

Administration defended its steps to thwart terrorism, but critics argued that the subversion of the Constitution could not be defended. The case of Yasser Hamdi, an American citizen declared an "enemy combatant" and held incommunicado in a Navy jail, is illustrative. A federal judge ruled that Hamdi had the right to an attorney. The essential issue was "whether the government could refuse to give U.S. citizens it says are enemy combatants the opportunity to tell their side of the story to any court, to a lawyer, or to the public." The result was that "assertive courts and congress insisted on more balance between liberty and security." KEY CHAPTER TERMS AND IDENTIFICATIONS civil liberties civil rights Bill of Rights--1791 Fifth Amendment Fourteenth Amendment Due process and equal protection clauses campus speech codes preferred freedoms--mostly derived from First Amendment partial versus complete incorporationists First Amendment freedoms--speech, religion, privacy, due process speech plus (symbolic actions) libel and slander, yelling fire in a crowded theater Communications Decency Act of 1996 Reno et al. v. ACLU et al. (1997) school prayer and "free exercise of religion" educational vouchers Zone of privacy Romer v. Evans (1996) due process rights 111

affirmative action suspect classifications--race and sex close scrutiny Plessy v. Ferguson Brown v. Board of Education University of California Regents v. Bakke City of Richmond v. Corson Adarand v. Pena Proposition 227 equality of opportunity equality of results Our Constitution is color-blind (Justice Harlan) civil libertarians Oncale v. Sundowner class action suits equity law private attorneys general American Civil Liberties Union NAACP Legal Defense & Educational Fund National Organization for Women Legal Defense and Education Fund test case landmark decision injunctions and orders Little Rock Central High School Miranda v. Arizona exclusionary rule Alien and Sedition Acts Civil Liberties and Terrorism Taliban and Due Process "enemy combatants" Yasser Hamdi

112

TEACHING SUGGESTIONS 1. Invite a representative of the ACLU to class. The function of the ACLU can be explained to the class, as well as a number of important cases involving civil liberties. 2. If possible, obtain a video of the documentary EYES ON THE PRIZE, which chronicles the civil rights struggle in this country. By showing it to the class, the full drama of the civil rights movement can be realized. 3. Ask students their opinions on whether prayer should be permitted in the public schools. Despite the prohibition against prayer, it is possible that students may have experienced religious activities in their high schools before coming to college. 4. Set up a debate involving the issue of the Internet and pornographic materials. As preparation for the debate, review the Supreme Court ruling (Reno). SUGGESTED RESEARCH ASSIGNMENTS 1. Study how other democracies approach issues of civil liberties and civil rights. How are freedom of speech laws different in Great Britain, for example, as compared with the U.S.? How are they similar? 2. Ask a team of students to interview local police officers regarding their use of the Miranda warnings in making arrests. In addition, what search and seizure procedures are they obligated to follow? The results of the interviews can be tabulated and form the core of a term paper. 3. Ask a few students to investigate the speech standards on their own campus. How have they evolved? How have they been applied? Are 113

students and faculty familiar with them? How much support is there for them at your college? MULTIPLE-CHOICE QUESTIONS 1. The main constitutional reason why the Bill of Rights has been incorporated relates directly to which specific amendment? a. Tenth Amendment b. Thirteenth Amendment c. Fourteenth Amendment d. First Amendment The equal protection clause prevents discrimination by private individuals when a. it is aided by state action such as a law. b. it involves a fundamental state interest such as education. c. both a and b are correct d. both a and b are incorrect The due process clause of the Fourteenth Amendment applies to a. the federal government only. b. only minority groups. c. all levels of government. d. laws passed since Miranda v. Arizona. The notion that a democratic society needs competition among ideas, as much as an economic marketplace needs competition, was stated by a. John Marshall. b. Oliver Wendell Holmes. c. Bill Clinton. d. Janet Reno.

2.

3.

4.

114

5.

According to the author, the most important civil liberties are those that a. protect ethnic minorities from the majority. b. protect children. c. protect players in the political game from government. d. all of the above The phrase equal protection of the laws is found in what amendment? a. the Fourteenth Amendment b. the Fifth Amendment c. the First Amendment d. none of the above The Supreme Court struck down the Communications Decency Act of 1996 because a. the technology creating the Internet was too new. b. the law was too vague. c. it did not protect children from viewing offensive material. d. a majority of the Court watch pornography. The Bill of Rights is fundamentally designed to protect a. individuals from other individuals. b. state governments from the federal government. c. individuals from government interference. d. all of the above The Supreme Court found a zone of privacy that shields individuals from government intrusion in a. the constitutional provision against illegal searches. b. the legal arguments behind the ruling in Plessy v. Ferguson. c. the First and Ninth Amendments. d. none of the above

6.

7.

8.

9.

115

10. In the 1996 decision of Romer v. Evans, the Supreme Court ruled a. a state could not single out a group and prohibit local laws designed to end discrimination against them. b. homosexual marriages should be on the same footing as heterosexual marriages. c. school prayer was constitutional. d. discrimination was permitted, as long as majority of the community disliked the discriminated group. 11. According to the Supreme Court, affirmative action programs must now a. have a compelling interest behind them. b. be narrowly tailored. c. both a and b are correct d. both a and b are incorrect 12. The main thrust of Proposition 227, passed in California in 1994, was a. limiting the real estate taxes in every urban community. b. ending the ACLUs support of illegal immigration. c. banning the use of race and sex in college admissions and public employment. d. mandating that race be a major factor in admitting students to public universities. 13. In Adarand v. Pena, the Supreme Court threw out a. the equal protection clause. b. federal set-aside programs for minority contractors. c. all forms of affirmative action. d. the exclusionary rule.

116

14. Which of the following cases dealt with the issue of sexual harassment? a. Miranda v. Arizona b. Oncale v. Sundowner c. Steelworkers v. Weber d. Plessy v. Ferguson 15. Protective legislation refers to a. laws outlawing the exploitation of child labor. b. laws regarding wage and hour regulations for female workers. c. laws passed to outlaw unsanitary work conditions. d. all of the above 16. The exclusionary rule refers to a. those cases that the Supreme Court will not hear. b. a principle upheld in Brown v. Board of Education. c. throwing out evidence obtained by unconstitutional means. d. an intent to withhold information from the courts. 17. Due Process rights are most closely associated with which amendments? a. First and Second Amendments. b. Fifth and Fourteenth Amendments. c. Sixth and Eighth Amendments. d. None of the above. 18. The ACLU a. defended individuals against the red scares of the 1920s. b. opposed civility codes on college campuses. c. a and b are correct d. a and b are incorrect

117

19. A landmark case is one that a. is heard by the Supreme Court. b. creates a new general standard for judicial decisions. c. becomes an amendment to the Constitution. d. Congress refers to the Supreme Court. 20. Thurgood Marshall is significant because he a. helped defend O.J. Simpson in his civil liberties case. b. was the only male employee in the National Organization for Women Legal Defense & Educational Fund. c. was the first black appointed to the Supreme Court. d. wrote the landmark Brown v. Board of Education decision. 21. When President Dwight D. Eisenhower sent federal troops to protect black students at the Little Rock Central High School, he was a. enforcing a court order. b. protecting the rights of the accused. c. adhering to provisions in the Equal Rights Amendment. d. stopping the Governor of Arkansas from filing an injunction. 22. If state agencies refuse to obey a Supreme Court order, the Supreme Court can a. do nothing, but the Chief Justice can appeal to the governor. b. fire the heads of those disobedient state agencies. c. ask the states national guard to close down those agencies. d. none of the above 23. The Dept. of Education permits which religious activities in schools? a. Student prayer by individuals or groups b. Reading the Bible c. Religious activities before or after school d. All of the above

118

24. The attitude of the Justice Department toward civil rights groups during the Clinton administration was a. generally hostile. b. generally supportive. c. one that preferred a class conscious approach. d. one preferring a color-blind approach. 25. Which of the following is a true statement about Yasser Hamdi? a. He was a Saudi citizen when captured. b. He was held incommunicado in a Navy jail. c. A federal judge in Norfolk ruled that he was not entitled to a lawyer. d. All of the above are true statements. 26. How many people were locked-up by the Bush Administration after 9/11 without the protection of civil liberties? a. 1200 b. 2000 c. 2500 d. 3000 27. Thomas Sowell argues that a. affirmative action does not work. b. affirmative action creates resentment against blacks. c. a and b are correct d. a and b are incorrect 28. Sexual harassment law is an example of a. freedom of action. b. an issue the Supreme Court will not address. c. President Clintons demand to mend it, but dont end it. d. protective legislation.

119

29. Which of the following is a characteristic of the Internet? a. It's global b. Its anonymous c. Both a and b d. Neither a nor b 30. O.J. Simpson refusing to testify in his criminal trial was an example of a. freedom of speech. b. civil rights. c. due process. d. freedom from harassment. TRUE-FALSE QUESTIONS 1. 2. 3. 4. 5. 6. 7. Civil liberties are those rights that protect citizens against government actions. Civil rights are the protections granted in the Constitution, recognizing that all citizens must be treated equally under the laws. The Supreme Court has stated that only federal legislation can address civil liberties issues. The Miranda decision ruled that suspects did not have to be notified prior to their arrest that they were entitled to an attorney. In the past few years, the courts and voters have reduced the impact of affirmative action. Members of the ACLU are among the most prominent supporters of First Amendment rights. John F. Kennedy used the army to enforce a federal court order. 120

8. 9.

Regarding segregation, the Supreme Court reversed itself in this century by rejecting the separate but equal doctrine. Since most employees of the Justice Department are career civil servants, it does not matter who the president is when issues of civil rights and liberties arise.

10. Wartime presidents have rarely tried to find a way around civil liberties. ESSAY QUESTIONS 1. Summarize the main philosophical differences between the partial incorporationists and the complete incorporationists. Then, do the same for judges who practice judicial activism versus those who believe in judicial restraint. 2. Do Supreme Court Justices follow the judicial philosophies expected of them by the presidents who nominate them? Why? 3. Should sex be considered a suspect classification? Should it be treated the same as race? Cite examples of where it has been treated the same and where it has been treated differently and argue your point of view. 4. Black conservatives argue that affirmative action hurts African Americans more than it helps them. What are the arguments for and against them? How has Proposition 227 affected the African-American community in California? 5. How have the courts restricted the rights of criminal defendants over the past few years?

121

6. Is it possible to keep the Internet separate from the freedom of speech restrictions that govern other areas of public communications? Chapter 6 Multiple Choice 1. C 2. C 6. A 7. B 11. C 12. C 16. C 17. B 21. A 22. D 26. A 27. C True-False 1. T 6. T 2. T 7. T 3. C 8. C 13. B 18. C 23. D 28. D 3. F 8. T 4. B 9. C 14. B 19. B 24. B 29. C 4. F 9. F 5. C 10. A 15. D 20. C 25. B 30. C 5. T 10. F

122

CHAPTER SEVEN Voters and Political Parties

CHAPTER OVERVIEW This chapter begins by examining the reasons for the growth of non-voting in America. Among these are social class, the political socialization process, electoral barriers (registration), and income and educational levels. Subjectively, there is the growing lack of political efficacy and citizens distrust of government. The historical development of American political parties and the two-party system is discussed, as are maintaining, deviating, and realigning elections. The arguments of whether or not we are currently undergoing a political realignment are covered as well. The functions, organization, and present viability of our two major political parties are then discussed. The chapter concludes with a case study on ECampaigning in the 2000 Election. LEARNING OBJECTIVES After reading chapter 7, the student should be able to: 1. Explain those factors that influence whether a citizen will vote or not vote, such as class, occupation, income, partisan loyalty, registration requirements, and the sense of political efficacy.

123

2. Define and explain the significance of the political socialization process. 3. Diagram the structure of the modern American political party. 4. Discuss the evolution of the American party system, from the Federalists to the Whigs to the Republicans and Democrats. 5. Distinguish among maintaining, deviating, and realigning elections. 6. Itemize the main functions of a political party. 7. Explain the factors that have promoted, and currently weaken, the twoparty system in America. 8. Discuss the arguments for and against a current presidential realignment. 9. Discuss the pros and cons of E-campaigning, using the 2000 election as a basis for that discussion. 10. Differentiate between hard and soft money and discuss the impact of the McCain-Feingold-Cochran campaign finance reform bill. OUTLINE I. Voters A. Who Votes. While citizens who are 18 or older and who have satisfied residency requirements are eligible to vote, voter turnout has declined in most recent presidential elections. The first significant increase since 1960 occurred in 1992, when 54 percent of the voting-age population voted, but voter turnout declined again in 1996. In the 2000 presidential election, 51.2 124

percent (105 million people) of the eligible population voted (up. 2.2 percent from 1996, but lower than the 62.8 percent who voted in 1960). In 1998, only 37.6 percent of eligible voters turned out. In 2002, only 39.3 percent of eligible voters went to the election booth. This turnout was still 20 percent lower than midterm elections in the 1960s. So, what influences people voting, and why has non-voting generally increased? B. Political Socialization. This is the process of learning political attitudes and behavior. Family background, schooling, peer groups, and ethnic and religious characteristics all influence the way people vote. Catholics vote Democratic more than Protestants and overwhelmingly favor aid to parochial schools; Jews and blacks tend to vote Democratic and are more supportive of social programs. Class and Voting. Class is defined as a groups occupation and income, as well as its awareness of its relation to other classes. Class is very important in shaping political attitudes. The three broad classes--upper, middle, and working--show differences in peoples political views; but these objective categories are really subjective, because most Americans, whatever their incomes, view themselves as being middle class. However, class, as it is reflected in education, income, and occupation, does influence peoples attitudes. For example, working-class people tend to be more conservative on civil liberties and more liberal on economic issues. Middle-class people show opposite tendencies in these two areas. Government policies and economic growth impact classes differently. During the prosperous 1990s, the income for 90% of families barely changed, compared to a $69,000 increase in average income for the top 1 percent of Americans, from 19961997. This increase was nearly triple the total average income of the bottom 90 percent. 125

C.

II.

Who Doesnt Vote A. Non-voting has grown in recent years and appears greater among the lower classes. Non-voters are less educated, non-white, rural, southern, poor, blue-collar, and very old or very young. Part of the reason for increased apathy derives from alienating events, like Watergate and Vietnam. More important has been the socialization of the working class and the American belief in a classless society, which discourages working-class issues and participation. Generally, the people with the biggest stakes in society are the most likely to go to the polls. B. Barriers. Electoral barriers and a lack of political efficacy have also been identified as reasons for low turnout among the poor and working classes. State registration laws often make voting inconvenient, but the 1993 Motor Voter legislation made registration easier, by allowing voters to register when they get their drivers licenses. Also, the six states that had same day registration witnessed higher voter turnouts. Still, in 1996, fewer people registered to vote and even fewer voted. By 1998, registration numbers increased, but voter turnout continued to decline.

C. Subjective Explanations. The category of non-votersmore than half of the American populationcovers a great deal of ground. While it is true that non-voters feel alienated from government, this is also true for voters. Since the mid-1960s, people have lost confidence in Washington. Indeed, today, three out of four people say they no longer trust governmentthe exact opposite from the early 1960s. Events such as Vietnam, Watergate, Iran-Contra, and Monica Lewinsky have contributed to a popular distaste for government. There remains an uncertainty about the causes of growing non-voting in America. The vote is still a powerful agent of change. For example, even

126

though Jesse Jackson was an unsuccessful presidential candidate, his two races increased the registration of new black voters. This, in turn, allowed the election of officials who were more sympathetic to the needs of the poor. Another example was the 1994 congressional elections, when GOP control of Congress led to new conservative approaches to policy. III. History of Political Parties A political party is an organization that supports candidates for public office under a specific label. The Constitution, by decentralizing and diffusing power, created a need for political parties to centralize and concentrate power. A. The Rise of Today's Parties: The Federalists and AntiFederalists were originally factions that formed because of disagreement over constitutional issues. They later developed into parties. The Federalists championed a strong national government, while the Anti-Federalists became the DemocraticRepublican party (led by Thomas Jefferson), supported the interests of the small farmer, and kept control of the government until 1828, when it split into two new parties, the Whigs and the Democrats. Under Andrew Jackson, the Democratic party became identified, at that time, with the common people. The Republican party, formed in 1854 as an antislavery party, first won the presidency with Abraham Lincoln. Maintaining, Deviating, and Realigning Elections. Elections are either maintaining (keeping party strength constant), deviating (showing a temporary shift in popular support for a party), or realigning (showing a permanent shift in support for and strength of a party). Between 1860 and 1932, the Republicans maintained control with only temporary deviations, for example, in the elections of Wilson, in 1912 and 1916. In 1932, there was a major realignment, in which the Democrats became the majority party under FDR. Some analysts feel that the 1980 127

B.

election may have reversed this alignment, but, even though Bushs 1988 win was solid, lower voter turnout and the Democrats control of Congress make a near-term pattern of partisan balance seem more likely than a realignment. Bill Clintons victories, in 1992 and 1996, made the argument for Republican realignment doubtful, while the GOPs control of Congress since 1994 (for the first time, a majority of southerners voted Republican for Congress) countered the Clinton effect. The GOP kept its majority in Congress through the 1996 and 1998 elections and, in 2000, the GOP won the presidency with George W. Bush, as well as control of both houses of Congress. In 2002, clear Republican majorities were obtained in both the House and Senate, as voters responded to a popular president, a united Republican party, and a Democratic party in disarray. However, political pundits were unsure as to whether the New Deal coalition was dead and whether the lack of strong party loyalties among independent voters made a realignment of the parties likely. C. Democrats versus Republicans versus Independents. There is a difference between party image and party reality. The image of the typical Republican (WASP, big business, law and order, etc.) and the typical Democrat (minority, working class, urban, etc.) are stereotypes, and the reality is more complex. Party leaders are more extreme in their views than are their moderate followers, and both parties are divided within by moderate and extreme wings. Independent voters are growing, as party ties weaken. The new independents are as politically astute as partisans, although there is a debate over whether they are really closet partisans (the myth of the independent voter). Party identification is still the strongest predictor of voter behavior, but a decline in partisanship has led to the 2001 poll trends, revealing that 40 percent of adults were independents, 34 percent Democrats, and 26 percent Republicans. In short, voters

128

are less attached to the two parties (rise of media, candidatecentered campaigns, divisive events, issue voting). D. Party Functions 1. To contest elections. The party takes positions on issues and gives people a basis for making a choice. It educates voters on the process and the issues, and recruits people to work directly in campaigns. To organize public opinion. The party provides a channel of communication for peoples ideas, and for voter approval or disapproval. To aggregate interests. The party organizes many special interests into larger coalitions under the party umbrella. To incorporate changes. The party responds to movements outside the major parties that show widespread support.

2. 3. 4.

IV. View From the Inside: Party Organizations A. MachinesOld and Modern. Parties are no longer strongest at local levels. Rigidly organized local parties (machines) have been weakened by the institution of public welfare agencies, civil service reforms, and direct primaries. An example of the old machine was Richard Daleys Democratic party organization in Chicago. It was based on ethnicity and patronage. Modern machines based on candidates, not parties, have used new voter-contact technology to prosper. Many members of Congress now maintain Leadership PACs that promote their ambitions for higher office through the raising of money. Finally, most recent White Houses have been charged with using campaign staffers to form modern political machines (i.e., the permanent campaign). American Party Structure. The pyramid structure of local, county, and state parties continues to underpin the national party 129

B.

organizations. Note that the job of out-party officials is to show that the party is still alive, through fundraising and voter registration drives. For the in-party, government officials use the party organization, not vice versa. Finally, state parties are usually stronger than local parties, due to their linkages with the national party structure, professional staffers, distribution of funds, and patronage. C. National Party Organization/Soft Money. Between elections, a national committee, which maintains a professional staff, governs each party. Recently, led by the Republicans, the national parties have been strengthened. The GOP has retained a fundraising advantage over the Democrats, although the gap has narrowed (in 2002, GOP party committees raised $527 million, the Democratic party committees, $344 million; previously, in the 1999-2000 election cycle, GOP party committees raised $692 million, compared to the Democratic partys $513 million). The GOP also holds a substantial lead in hard money contributions, compared to the Democrats, due to their large pool of small donors ($447 million compared to $270 million). The GOP funding base also has produced party loyalty in Congress and the ability to buy expensive campaign technology. Much of the party activity at the state and local levels is supported by soft money, which includes unregulated contributions that are supposed to be used for party-building and get-out-the-vote activities. The national party committees raise soft money from wealthy individuals, corporations, and unions. These large donations are very controversial and led directly to the 1996 fundraising scandals. In response, the McCainFeingold-Cochran Campaign Reform Bill, banning all soft money contributions to national political parties and phony advocacy ads run by corporations and unions around election time, was introduced in the 107th Congress. It eventually became

130

law, in early 2002. However, both parties were seeking new ways to collect soft money contributions. D. The National Convention. National conventions are held every four years. The delegates, chosen by state party organizations in various ways (70 percent chosen now through primary elections), adopt a platform, a statement on issues used to distinguish the partys position from that of the opposition. Individual planks (issues) in a platform can be controversial during a convention, such as GOP women, in 2000, pushing (unsuccessfully) for a plank supporting abortion rights. By a simple majority vote, the delegates elect their nominee for president. In recent decades, the presidential nominee has been chosen on the first ballot. The nominee usually chooses a vice president who will balance the ticket. A good example was the Dukakis (Mass.) choice of Bentsen (Texas), in 1988. In 2000, the vice-presidential nominees were aimed at bolstering the weaknesses of the presidential nominees (i.e., Cheneys experience in the federal government and Liebermans antiClinton stand on moral issues).

V.

View From the Outside: The Two-Party System The United States has a two-party system nationally, but locally, various arrangements have existed. The solid (Democratic) South traditionally had, until recently, a one-party system, for example, and Minnesota is said to have a three-party system (Ventura and the Reform party). A. Causes of the Two-Party System 1. Historic dualism. The original Federalist/Anti-Federalist division established the tradition of two-party domination in the country.

131

2. 3. 4.

Moderate electorate. Did the system make voters moderate, or vice versa? Still, Americans do not like extreme political choices. Structure of our electoral system. Plurality penalizes minority parties. By contrast, proportional representation ensures minority parties at least some representation. The adaptability of the two major parties. This allows for the absorption of third parties and their ideas. Thus, the Democrats co-opted organized labor from the Socialists. Al Gore tried to win votes from Ralph Nader, in 2000, by stressing populist themes and charging the GOP with being the party of the very wealthy.

B.

The End of the Two-Party System? The diminishing loyalty of voters to either political party and the rise of sentiment for third parties has led to a questioning of the viability of the two-party system. The two-party system has prevented the country from being polarized, or severely divided. Yet, despite Republican victories in 2000 and 2002, the two parties remain close to being equal, and, as a consequence, are often deadlocked.

VI. Case Study: E-Campaigning in the 2000 Election In the 2000 election, the Internet proved valuable in the areas of fundraising, voter contact and advertising, and research. In terms of fundraising, Senator McCain raised $6 million from online sources (in the 2000 election, up to 10% of all campaign funds were derived from Internet activity). The Internet was faster and more costeffective than either mail or phones (Internet fundraising costs about 8 cents per dollar, compared to 50 cents by phone, 70 cents by mail). Regarding campaign content, both Bush and Gore used the Web, from allowing downloading of position papers to including video footage of campaign ads and online polling. As the author puts it, the great advantage of the Web was that it allowed candidates to lay out their positions on issues without space or time limits. 132

Conversely, Internet ads were not widespread in 2000, due to uncertainty about its impact (compared to TV or mail) and concern that such e-mail ads would antagonize voters. Virtual Grassroots Campaigning, in 2000, was expanded, with both parties establishing thousands of e-mail precincts around the nation. Both sides used their sites to bash each other, with the GOPs National Committee site being called Gorewillsayanything.com! E-mail for campaigning is cheap and can be done instantly. However, it requires e-mail addresses of voters, who in turn can assist in getting out the vote (Jesse Ventura used e-mail in his 1998 election for Governor, stamping out rumors that he favored legalized prostitution; Senator McCain recruited 26,000 online volunteers). In conclusion, much of the e-campaign lacks human emotion and personal contact, but further advances in technology may rectify this shortcoming. KEY CHAPTER TERMS AND IDENTIFICATIONS voter turnout political socialization social class--working, middle, and upper class as reflected in education, income, occupation "people with biggest stakes in society" go to the polls Hispanic voting is low Motor Voter legislation electoral barriers to votingregistration, residency, time constraints nonvoters (half or more of the population) political efficacy; lack of trust in government political apathy Jesse Jackson subjective explanations for non-voting political party--centralizes power Parties--contest elections, organize opinion, aggregate interests Third parties; parties incorporate changes 133

Federalists versus Anti-Federalists Democrats and Whigs Republican Party; Lincoln as a "dark horse" GOP--"Grand Old Party" The "New Deal" coalition maintaining, deviating, realigning elections dealignment Ross Perot, Ralph Nader 2000 election--George W. Bush 2002-GOP control of Congress independent voters the typical Republican the typical Democrat party followers versus leaders liberals versus conservatives strong and weak partisan voters independent voters; decline in partisanship political machines (Richard Daley in Chicago) leadership PACs, GOPAC (Newt Gingrich) the permanent campaign pyramid-shaped party structure in-party; out-party state parties/committees national party committee/chairperson professional political staff soft money (now banned by McCain-Feingold) McCain-Feingold-Cochran Campaign Reform Law hard money (regulated funds for federal candidates) the Keating Five national convention party platform and individual planks presidential primaries and caucuses balance the ticket one-party, two-party, multiparty systems historic dualism; moderate views of the voter 134

single-member district/plurality proportional representation adoption of third-party programs by two main parties prevention of polarization "Seinfeld elections" (about nothing!) E-Campaigning, online donations Jesse Ventura and Senator John McCain Gorewillsayanything.com TEACHING SUGGESTIONS 1. Ask the class if American voters should be required to vote. Would a fine for not voting increase voting in America--why or why not? Second, ask for a show of hands on how many members of the class are actually registered to vote (those students 18 or older). For those who are not, probe more deeply as to the reasons why. 2. Discuss with members of the class their earliest memories of political issues. Did parents discuss politicians and issues? Also, are the students political party affiliations the same as their parents? An informal poll could prove interesting regarding the political socialization process. 3. What should be the role of money in politics? Will McCain-Feingold improve the integrity of the political process in America? Also, how does the high cost of campaigns hurt the potential for third-party organizations? 4. Ask students if they have ever used political websites to find out more about American politics. If some have, then ask which sites have been used and why they are helpful (this could also take the form of a research assignment).

135

SUGGESTED RESEARCH ASSIGNMENTS 1. Examine why people vote the way they do. Have students do some informal polling to examine how people decide what presidential candidate to vote for. Are parties, the individuals, or issues most important in the decision? How important was what they heard during the campaign in making up their minds? 2. Assign several students to examine which issues influenced the outcome of the 2000 presidential election. Did the students interviewed mention the impact of these issues on how they decided to vote? How did both political parties try to package Al Gore and George W. Bush? Be sure the students cover the presidential TV debates. Finally, examine the issues during the 2002 midterm elections and ask students why the GOP did so well. 3. Study the current political functions of the national nominating conventions. Why has network television coverage of those conventions declined in recent years? 4. How has the Motor Voter legislation worked in your locality? Have students interview people registering new voters to get a sense of the change that this reform has brought. MULTIPLE-CHOICE QUESTIONS 1. Class identification in the United States is a. constant. b. weak. c. strong. d. irrelevant.

136

2.

Motor Voter legislation a. allows voters to register when they get their drivers licenses. b. was directly responsible for the increase in voter turnout in 1992. c. was struck down as unconstitutional. d. a and b only Non-voters who feel it wont make any difference act a. from a sense of arrogance. b. from a lack of political efficacy. c. due to a lack of partisanship. d. from a strong sense of class consciousness. Trust in government a. rose when the Congress impeached President Clinton. b. is identified by non-voters as the main reason for not voting. c. has declined since the 1960s. d. all of the above Which of the following is learned by children in the family regarding political attitudes, beliefs, or values? a. the political party preferred by their parents b. the advantages of a democracy c. pride in the nation d. all of the above Jesse Jackson used his two presidential bids to a. register new black voters. b. bargain with Democratic leaders on issues concerning poor Americans. c. alienate voters who opposed his liberal message. d. both a and b are correct

3.

4.

5.

6.

137

7.

The best predictor of how a person will vote is his or her a. social class. b. income. c. party identification. d. age. Who was the founder of the Democratic-Republican party? a. George Washington b. Thomas Jefferson c. Alexander Hamilton d. Andrew Jackson Which of the following is a false statement about American voters? a. over three-quarters know the name of their representative in Congress b. most are well-informed about politics in general c. the majority engages in a wide variety of political activities d. all of the above are false

8.

9.

10. Which of the following was not part of the New Deal Coalition? a. eastern liberals b. white Southerners c. upper-class WASPs d. union members 11. The early Democratic party of 1828 was led by whom? a. Alexander Hamilton b Abraham Lincoln c. Thomas Jefferson d. Andrew Jackson

138

12. The typical Republican voter a. supports limited government. b. supports affirmative action. c. belongs to a labor union. d. none of the above 13. According to the chapter, a party _____ political power. a. polarizes b. centralizes c. decentralizes d. balances 14. American political parties a. organize public opinion b. put together coalitions of different interests c. demand party loyalty on all issues d. a and b only 15. The two Republican elections of Dwight Eisenhower in 1952 and 1956 were examples of _____ elections. a. maintaining b. dealigning c. deviating d. realigning 16. Leadership PACs a. promote party interests across the country. b. lost their effectiveness with the resignation of Speaker Newt Gingrich. c. a and b are incorrect d. a and b are correct

139

17. On social programs, Jewish voters a. vote like middle-class Protestant voters. b. vote like black voters. c. are the most independent voters. d. support candidates promoting an American First agenda. 18. The newer style of political machines a. are centered on candidates, not party. b. is not as focused on ethnic group identity. c. uses the mass media, rather than personal contacts. d. all of the above 19. The Whig party joined others to form the _____ party. a. Republican b. Democratic-Republican c. Federalist d. Socialist 20. The national party committees a. have representatives from each state party organization. b. maintain large professional staffs. c. raise money from special interests. d. all of the above 21. Richard Daleys political machine was located in the city of a. Chicago. b. New York. c. San Francisco. d. Boston. 22. Soft money a. helps fund state and local parties. b. is of no consequence after McCain-Feingold. c. means the same as "hard money." d. none of the above 140

23. What is now the single most important source of money in American politics? a. federal taxes b. oil companies c. the Republican party d. the Treasury Department 24. The bottom level of the typical state party organization is termed the a. congressional district committee. b. war or precinct level. c. county committee level. d. state committee. 25. According to the chapters case study, E-Campaigning a. is relatively useless in obtaining campaign research. b. was shunned by Senator John McCain. c. was especially effective in conveying the emotion inherent in politics. d. was more cost-effective that either direct mail or telemarketing. 26. Thomas E. Patterson has argued that a. media coverage of elections emphasizes the horse race too much. b. balancing the presidential ticket leads to defeat. c. the party platform is essential to understanding politics. d. balancing the presidential ticket is essential for victory. 27. Which of the following is associated with proportional representation? a. single-party political systems b. multiple-party political systems c. two-party systems d. authoritarian political systems that have no political parties

141

28. The myth of the independent voter refers to this: a. the non-existence of independent voters b. that most of them follow politics closely c. that most are really undeclared partisans d. that most of them dont follow politics closely 29. The two vice presidential nominees in 2000 were a. Lieberman and Cheney. b. Kemp and Gore. c. Mondale and Quayle. d. none of the above. 30. The New Deal Coalition has been weakened by a. the rise of independent voters. b. the strength of the Republicans in the South. c. the 1994 elections. d. all of the above TRUE-FALSE QUESTIONS 1. 2. 3. 4. Throughout American history, there have always been many active and successful parties. Most Americans view themselves as members of the middle class. Non-voters tend not to be as wealthy as voters. Motor Voter has been an unqualified success.

5. Voter turnout is greater in the U.S. than in any other democracy in the world. 6. The Keating Five illustrates the problem of wealthy individuals trying to buy influence through campaign contributions. 142

7. 8. 9.

Single-member districts are an important support for the two-party system. Fundraising online actually decreased in 2000, when compared to 1996. The myth of the independent voter is that they are as politically knowledgeable as partisan voters.

10. Democratic and Republican party followers often disagree more with each other than with their party leaders. ESSAY QUESTIONS 1. Political parties have in some ways strengthened, and in some ways weakened, in recent years. Give examples of both trends. Why, in your opinion, did this happen? 2. Imagine that you are a giving a speech to a group of poor people-welfare mothers, homeless, etc.-- explaining why it is important for individuals to vote. What would you say to them? 3. Imagine that you are asking business leaders to contribute money to support your campaign for Congress. How would you ask? What would you promise them? 4 Are political campaigns too expensive? How can one balance the First Amendment with limiting the influence of wealthy political contributors?

5. Study a country that maintains a multi-party system. What is gained by having many parties? What is lost? Will the U.S. ever adopt that form of politics? 143

6. How do political parties in Africa and Asia differ from those in America? 7. Specify which political beliefs are likely to be preferred by liberals or conservatives? 8. Do you consider yourself an independent voter? Why or why not? 9. What recommendations can you think of to increase voter turnout? Conversely, how could you argue that apathy is good for the U.S. political system? 10. What is meant by both old and new party machines? Chapter 7 Multiple Choice 1. B 2. A 6. D 7. C 11. D 12. A 16. C 17. B 21. A 22. A 26. A 27. B True-False 1. F 6. T 2. T 7. T 3. B 8. B 13. B 18. D 23. C 28. C 3. T 8. F 4. C 9. D 14. D 19. A 24. B 29. A 4. F 9. F 5. D 10. C 15. C 20. D 25. D 30. D 5. F 10. F

144

CHAPTER EIGHT Interest Groups and the Media

CHAPTER OVERVIEW This chapter explores how interest groups and the mass media affect American politics. Various types of interest groups and tactics of influence are covered, along with the role of lobbyists. Another important theme deals with money and its relationship to political campaigns, as shown by the power of PACs. Reform proposals to curb the power of PACs are also noted. The importance of the mass medias relationship to national politics comprises the other half of the chapter. Television and newspapers are emphasized. News coverage and the significance of media mergers and new media are discussed. The three major types of media messages--news reports, entertainment programs, and advertising--are explained. An important sub-theme, media profits and information diversity, follows. The interaction of politicians and media is examined, and the chapter ends with a case study of a fictional day in the life of a candidate for the U.S. Senate. LEARNING OBJECTIVES After reading chapter 8, the student should be able to: 1. List the various types of economic interest groups, with specific examples of each type. 145

2. Review the roles of the lobbyist and the various kinds of lobbying tactics. 3. Discuss the relationship between campaign contributions and PACs (Political Action Committees). 4. Summarize how the media affects political life, specifically noting such sub-themes as media mergers, sound bites, media messages and profitability, news management, media campaign ads, and the medias relationship to public opinion. 5. Discuss the political importance of the rise of new media, like the Internet, and alternative media forums, such as late-night TV talk shows. 6. Summarize the key points found in the chapter case study, The Candidate: A Day in the Life . . . . OUTLINE I. Interest Groups Interest groups organize to pursue a common interest, by applying pressure on the political process. There are numerous groups in America, as Alexis de Tocqueville noted in 1835. Interest groups are usually more tightly organized than political parties. They are financed by contributions or by dues-paying members. A. Types of Interest Groups. The most important and widespread type of association is based on common economic interests. Business groups (Chamber of Commerce, NAM), including corporations, are interested in profits and supporting a system that maintains them; professional associations (teachers, doctors, lawyers, etc.), labor unions (AFL-CIO), and agricultural lobbies 146

are among the most powerful groups seeking protection and favors. In addition, there are religious (Christian Coalition), racial, environmental (Sierra Club), public interest, and political groups, all seeking to influence government policies. B. Lobbying. Interest groups or individuals put pressure on the government to act in their favor. The typical lobby is a professional staff of experienced people who provide information to congressional committees and the bureaucracy (the law requires Washingtons lobbyists to register and limits gifts and meals for congressmen to $100 a year, from any one person). These lobbyists include former members of Congress or the executive branch. Note the example of Senator Phil Gramm of Texas, who, after retirement from Congress, joined the Wall Street firm of UBS Warburg, for an annual salary of over $1 million. Indirect lobbying (used by the NRA) may involve campaigns to enlist public recognition and support (grassroots campaigns--AARP) and attempts to influence other interest groups for their cause. In some cases, groups will form a coalition to lobby for one specific goal (e.g., free trade). Direct lobbying usually takes place in congressional committees and executive bureaucracies. Lobbyists, bureaucrats, and congressional committees comprise the so-called Iron Triangle. Advocacy ads do not fall under campaign financing rules (they dont directly support a candidate and are not coordinated by campaign officials), so they can help a candidate and avoid restrictions on raising/spending money in an election. John McCain found this out when TV stations began broadcasting ads in key primary states (2000 election), attacking his record on the environment. These ads were sponsored by two Texas billionaires (Sam and Charles Wyly) who were Bush supporters. C. Campaign Contributions and PACs. The most controversial aspects of lobbying relate to campaigns and elections. Interest 147

groups can support friendly candidates or work to defeat candidates they oppose (in the 2000 election, energy, tobacco, and high tech companies supported Bush). The threat of using money against an incumbent can also have an impact upon decision-making. Political action committees (PACs) are set up by private groups in order to influence elections. (The earliest PAC, created in 1955, was the AFL-CIOs COPE.) Since campaign reforms of the mid-1970s put limits on individual donations, the numbers of PACs have mushroomed (from 608 in 1975 to 4,393 in 2000). As a result, election funds from special interests, especially business groups, have grown enormously, the exact opposite intention of the reforms. Money does buy access, the right to talk to a legislator (by 2000, GOP candidates got $86 million from PACs, while Democrats got nearly $80 million). The 2000 congressional elections were the most expensive in history. To win an open Senate seat cost almost $14 million, and winning House candidates spent $1.3 million. However, candidates who spend the most do not always win (Charles Schumer). Most recent attempts at new reforms havent offered enough enticements to both in and out politicians to secure passage. Various attempts to reform PACs are usually opposed by one party, which feels the proposed reforms would benefit the other party. D. Do Group Interests Overwhelm the Public Interest? The growth in numbers and influence of interest groups is unarguable (by mid-1999, there were more than 38 registered lobbyists and $2.7 million in lobbying spent for every member of Congress). The failure of reforms is equally evident. As the number of interest groups has multiplied, the result has been hyperpluralismtoo many groups making too many demands on government (see Greiders Grand Bazaar). These criticisms are overstated. Special interests do not dominate the entire game. They are countered by politicians, other groups, and the media, all of which limit an interest groups power and ensure 148

that the results of the political competition remain unpredictable. Note that many studies reveal that lobbyists have little success in persuading members of Congress to change their minds; but "lobbyists gain support by presenting their cause as consistent with a popular concern." II. The Media (sometimes called the fourth branch of government) A. What Are the Media? Media includes those means of communication that permit messages to be made public. Television dominates the mass media and is, in turn, dominated by the three major networks, CBS, NBC, and ABC. These networks function as agencies that produce and sell programs with advertising to their affiliates (each network had over 200; Fox had 177). In recent years, the three networks have been challenged by Fox and various cable stations (65 million television households get their signals through cable). Cableonly stations such as CNN and C-SPAN have also become important. New technologies are causing the distinctions between television, computers, and telephones to be blurred. For example, the Internet provides endless sources of information and entertainment. By 2002, the Internet was used by over twothirds of the adult population in America. Newspapers, though more varied, have shown a rapid decline in number and competitiveness in this country. At the turn of the century, there were 2,226 daily papers in the United States. By 2000, there were only 1,647. In 1920, there were 700 cities with competing newspapers. Today there are only 13. What Do the Media Do? The media provide three major types of messages--news reports, entertainment programs, and advertising. In news reports, the media selectively supply accounts of the most important events and issues (however, note that few people pay much attention to political news, especially young people under the age of 30). The most important function 149

B.

the media performs is agenda-settingputting together an agenda of national priorities (what should be taken seriously, lightly, or ignored). In short, the media cant tell people what to think, but it can tell them what to think about. Entertainment programs subtly give images of normal behavior and turn political conflicts into personal problems. Advertising also presents images of a level of material well-being presumed to be within the reach of most people. III. Media and the Marketplace of Ideas A. The Media and Free Speech. Although the Framers of the Constitution saw communications media as part of a free market of varied ideas, recent trends have countered this goal. Profitability has led to a decline in the number of competing newspapers, the increase in media chains, and the control of TV and radio by a small number of corporations (ABC-Capital Cities was acquired by Disney). More than 80 percent of the nations newspapers are owned by chains, and only three corporations own most of the nations 11,000 magazines. Market calculations have caused mergers, meaning that ownership of almost all major outlets are concentrated in a few corporate hands (Disney/ABC, Viacom/CBS, America Online/Time Warner). The dependence on advertising has led the media to avoid controversy, so as not to antagonize either consumers or advertisers. While the quantity of political information has certainly increased, the need for profitability and to provide entertainment has probably led to a decline in serious discussions of issues. The Media and the Government. These influence one another in various ways (in the early years of the Republic, the press was consciously partisan; by the end of the century, advertisers bankrolled papers). 150

B.

1. The Federal Communication Commission regulates TV and radio (since 1934). Radio and TV stations must renew their broadcast licenses every six years. 2. The media are both opportunity and adversary for campaigning politicians. Media advertising has accounted for 60 percent of campaign money in presidential races. Also, the quality of political news has not necessarily improvedthe typical sound bite has declined from 42 seconds, in 1968, to 7 seconds. Finally, even late night talk shows played a large role in providing information on the 2000 presidential election (Leno and Letterman). 3. Informal pressure by political leaders, through news management and promoting their favorable image to the public, is widespread. Presidents try to get on the good side of the media by giving favored reporters exclusive leaks of information, or through press conferences. Presidents now have large staffs of media experts and speech-writers to perfect their images. Bill Clintons attempts at good relations with the press came to an end when the media became fixated on covering the Presidents affair with a young intern. Clinton also pursued the alternative-media strategy, avoiding White House reporters in favor of local news anchors and televised town meetings. George W. Bush copied the Clinton strategy, adding a few twists of his own. He favored less-informed, less-critical reporters from outside of Washington and depended upon Dick Cheney for exploring issues in greater substantive depth. Yet, Bush's ability as a public communicator remained a presidential weakness. C. The Media and the Public. The media has a powerful influence on the political attitudes and actions of Americans. The media has the power to define alternatives. While they may check the activities of those in power, mainly they reflect and enhance the influence of the most powerful players in the political game. 151

IV. Case Study: Candidate: A Day in the Life . . . . . . A. Morning. She is running for the Senate. Her typically early morning is dedicated to two good-sized towns that she hopes will provide exposure, personal contact, and money to a candidate whose advisers still disagree over whether she should carry a purse on the campaign trail. She meets local teachers for breakfast, parries a reporters question about her children, and has a good press conference. She does two press conferences a day, each with a new position paper, scheduled with hopes for space in the morning or evening papers, or the big prize, evening TV news. Afternoon. Shes late to lunch at a senior citizens center (the volunteer college driver got lost), and she hurries back to her motel to ask for more money from two well-off contributors, to keep several effective ads on the air as the campaign winds down. More phone calls for more money are followed by filming of a TV spot--45 seconds on the screen, two grueling hours to film. Then its off to two coffees given by supporters. Evening. She and her advisors discuss fine-tuning her aggressive attacks on her opponent, which are cutting into his lead but raising her negatives. She postpones a decision to take advantage of her opponents sons activities -- which may be unsavory -- as a campaign issue and goes to a TV debate, the last event of the day. Having scored well on her opponent in the debate, she calls her husband and children at home for support and leaves word for another 5 a.m. wakeup call. Analysis. Todays candidate spends more time on media exposure and fundraising than personal contact with voters; paradoxically, the core value that appeals to news media and to potential givers is personal contact. 152

B.

C.

D.

KEY CHAPTER TERMS AND IDENTIFICATIONS "Special Interests" Alexis de Tocquevilles Democracy in America Interest group James Madison and The Federalist Papers economic interest groups business, professional, labor, agricultural groups National Association of Manufacturers; Chamber of Commerce American Medical Association and American Bar Association AFL-CIO American Farm Bureau Federation; National Grange Sierra Club, Greenpeace, Common Cause lobbying and lobbyists, Five Commandments of Lobbying 1995 lobbying reform billlobbyists must register Oregon U.S. Senator Bob Packwood Former U.S. Senator Phil Gramm's Wall Street position Christian Coalition Direct/Indirect Lobbying Iron Triangle grassroots campaigns coalitions John McCain and advocacy ads Sam and Charles Wyly AARP National Rifle Association Bushs corporate donors PACs (Political Action Committees); COPE Jon Corzine, Al DAmato vs. Charles Schumer "money buys access" "money as a threat and offer" hyperpluralism, Grand Bazaar of deal making Lobbyists' influence millionaires buying elections 153

Buckley v. Valeo the media--the fourth branch of government TV networks and affiliates--CBS, NBC, ABC, FOX CNN, C-SPAN Internet/World Wide Web Matt Drudge, Monica Lewinsky, Kenneth Starr Newspapers/wire servicesAP, UPI media messages--news, entertainment, advertising media selectivity medias agenda-setting function media outlets and profit need Neil Postman (media critic) marketplace of ideas chain ownership of media outlets media mergers--Disney/ABC, NBC/General Electric, etc. Penny papers Federal Communications Commission sound bites/pseudoevents late-night talk showsJay Leno, David Letterman Reagan as The Great Communicator; Lesley Stahls commentary leaks Clinton and Monica Lewinsky sex scandal Bush as a communicator/Vice-President Cheney President Kennedy and Pamela Turnure presidential press conferences on message E.E. Schattschneiders definition of alternatives medias political functions Federal Trade Commissions report on entertainment industry

154

TEACHING SUGGESTIONS 1. If possible, obtain videotapes of television political ads that were used in previous presidential campaigns. Show these clips to the class. Ask for reactions and discuss what messages were conveyed by those ads. 2. Discuss the role that lobbyists and interest groups play in American politics. Why are some groups more successful than others? Talk about the ethics of retired members of Congress lobbying their former colleagues. 3. Have students debate the question as to whether the many new media mergers of the past few years pose a threat to the free and diverse expression of ideas in America. 4. Ask students why so many young people (those under 30) are disinterested in politics. Also, how many members of the class watch an evening news program at least three times a week or read a newspaper editorial/column as least once a week? 5. Ask the class to consider the following questiondo millionaires entering politics have a guaranteed advantage over their opponents, based upon the historical recordwhy or why not? SUGGESTED RESEARCH ASSIGNMENTS 1. Ask selected students to correspond with some of the major interest groups in America such as the AMA, AFL-CIO, the NRA, the NEA, NOW, and so forth. Information can then be collected and analyzed from group brochures, pamphlets, flyers, etc. The purpose would be to summarize the political stands of these groups and to see how they promote their own interests.

155

2. Have your students pick an important issue form the recent past. Trace how that issue gained prominence. Did the media set the agenda by reporting on the issue, or did the politicians create a media stir by discussing the issues? How do these two groups interact to set the political agenda? 3. Have students talk with roommates and friends about what news items they recall from this same week. How much do people not assigned to watch the news recall about a weeks worth of news? Can one draw any conclusions about how attentive people are to the medias messages? 4. Ask several students to research how the media reacted to the three televised presidential debates during the 2000 election. Did the media appear to favor Gore or Bush? 5. Ask two teams of students to debate the following questiondoes money corrupt the American political process or not, especially after the passage of McCain-Feingold in 2002? MULTIPLE-CHOICE QUESTIONS 1. How much is spent in lobbying, per member of Congress? a. $1.7 million b. $2.7 million c. $3.7 million d. $4.7 million Which of the following groups is not classified as a business interest group? a. AFL-CIO b. Chamber of Commerce c. AT&T d. United States Steel 156

2.

3.

The Iron Triangle is said to comprise a. parties, interest groups, and the courts. b. lobbyists, bureaucrats, and congressional committees. c. lobbyists, courts, and bureaucrats. d. media representatives, bureaucrats, and congressional committees.

4. Interest groups a. became important in the U.S. during the 1970s. b. were identified by Woodrow Wilson in The Federalist Papers. c. represent only well-heeled corporate interests. d. have always been active, as evidenced by Alexis de Tocqueville in Democracy in America. 5. Which of the following groups is a professional interest group? a. Microsoft b. American Medical Association c. Associated Milk Producers, Inc. d. NAACP The fourth branch of government refers to a. the Congress. b. political parties. c. the media. d. interest groups.

6.

7. Which of the following is not an agricultural interest group? a. AMP b. SCLC c. NAACP d. All of the above are not agricultural groups.

157

8.

Top PAC contributors to Democrats are dominated by a. delivery services. b. womens groups. c. unions. d. pro-Israel groups. When the television networks stress the issue of environmental pollution for a week or so, instead of universal health care, they are performing the important media function of a. agenda-setting. b. sound-bites. c. entertainment. d. none of the above

9.

10. The growth of PACs a. has made politics cleaner and fairer. b. was an unanticipated consequence of campaign reform. c. forced big money contributors to donate illegally. d. is a new phenomenon directly linked to Republican control of Congress. 11. Nobody really knows what political contributions buy, but at the least they buy a. a speech in the donors favor. b. a vote in the legislature favoring the donors interest group. c. access to the politician. d. a job in the government for the donor. 12. The first PAC belonged to a. General Electric. b. the American Association of Retired Persons. c. the AFL-CIO. d. none of the above

158

13. As discussed in the chapters case study, advocacy ads a. do not advocate the election or defeat of a candidate. b. are funded directly from the treasury of corporations and labor unions. c. both a and b are incorrect d. both a and b are correct 14. PAC contributions typically a. mean that you are going to win the election. b. go to incumbent candidates of both parties. c. go to the challenger to promote more competitive elections. d. none of the above 15. E. E. Schattschneider meant, by his term definition of alternatives, a. the ability to set limits on political debates. b. the clarification of what is politically important. c. the making of certain political solutions reasonable and acceptable and others not. d. all of the above 16. President Clinton did not like press conferences because a. the press asked questions about their agenda and not his. b. the press mainly asked about scandals. c. the press asked questions about policies with which he was unfamiliar. d. a and b only 17. Interest groups support administrative agencies by a. providing information on their industry or group. b. contributing to re-election campaigns. c. offering personal and financial favors. d. all of the above

159

18. Which of the following is not a threat to the television networks? a. the Internet b. cable television c. newspapers d. CNN and C-SPAN 19. In its early days, the Lewinsky affair a. demonstrated the political role of the Internet. b. was first reported on by quasi-reporter Matt Drudge. c. was ignored because the New York Times did not break the story. d. a and b are correct 20. Television advertisers tend to prefer programs that are a. intellectually thought-provoking. b. conventional and inoffensive. c. controversial and challenging. d. overly concerned with factual information. 21. The decline in newspaper circulation a. is an outgrowth of high newsprint prices. b. does not concern political observers. c. both a and b are true d. both a and b are false 22. Some media organizations benefited from government action because a. the FCC generally restricted entry to competitors in radio and TV. b. they are exempt from government laws and anti-trust regulations. c. every agency develops free information for the press through their public affairs divisions. d. all of the above

160

23. Which of the following is not part of an Administrations attempt at news management? a. offering favored reporters exclusive leaks of information b. allowing the President or his staff to be caught in compromising positions c. presenting issues in a way that looks best for the President d. having the President deliver good news in person 24. Which of the following techniques illustrates a grass roots campaign? a. letter-writing campaigns b. an interest group forming a coalition with other groups c. a voter phoning her senator d. all of the above 25. Who wrote that Americans of all ages, all conditions, and all dispositions constantly form associations? a. Alexis de Tocqueville b. Tip ONeill c. Ronald Reagan d. Theodore Roosevelt 26. Bill Clintons alternative media strategy a. meant calling New York Times reporters at home. b. accelerated after the Lewinsky scandal broke. c. meant granting interviews to local news anchors and holding televised town meetings. d. meant holding online press conferences. 27. The purpose of grassroots lobbying is a. to raise more funds than the opposition. b. to get constituents to contact their own representatives. c. to form a national coalition of different groups. d. to prevent rash legislation from being passed.

161

28. Too many groups making too many demands on government is called a. the Democratic party. b. pluralism. c. hyperpluralism. d. lobbying. 29. The Internet was begun by a. the U.S. military. b. Apple Computer. c. Microsoft. d. independent computer hackers. 30. While Bill Clinton had the Monica Lewinsky scandal, Pamela Turnure was a name associated with a potential sex scandal involving a. Lyndon Johnson b. John F. Kennedy c. Jimmy Carter d. none of the above TRUE-FALSE QUESTIONS 1. 2. 3. 4. 5. Unlike political parties, interest groups do not compete for public office. Nearly 90% of the money in presidential campaigns goes for advertising. In 2002, the Internet was used by two-thirds of the adult population of America. There are far more public interest PACs than business PACs. Unions overwhelmingly support Democratic candidates through their PAC contributions. 162

6. 7. 8.

Only business PACs and corporations have an impact on governmental decision-making. Lesley Stahls media piece on Ronald Reagan was met by vehement criticism from the White House. The media is part of the Iron Triangle.

9. Over the years, sound-bites on TV have increased in length. 10. In 2000, late-night TV talk show hosts, such as Leno and Letterman, had a major impact upon political information obtained by young adults. ESSAY QUESTIONS 1. Why has grassroots lobbying increased in importance? Does this increase make the system more democratic or less democratic? 2. Does private money dominate Washington politics? Give examples that tend to support an affirmative as well as a negative answer. 3. Assess the impact of C-SPAN on Congress. Do members act more dignified or less so in front of the cameras? How was C-SPAN used by the Republicans to help them capture the Congress? 4. How does the medias need to be entertaining, to gain the largest possible audience, interfere with their political function as a channel of communication? Give specific examples. 5. In the chapter case study, our fictional candidate for the U.S. Senate spends most of her time preoccupied with media exposure and fundraising. In a well-reasoned essay, speculate as to how modern campaigning for Congress would change if television stations were 163

compelled under FCC regulations to furnish a much larger percentage of air time, at no cost to candidates. 6. How will the Internet affect American politics into the next century? Will it be an integral technology for candidate campaigns, grassroots campaigns, fundraising, and news-gathering? Can or will the government attempt to regulate this form of media? 7. What are the five commandments of lobbying? 8. Is the private sex life of an American president a proper concern of the media? Why or why not? 9. What is the significance of the Christian Coalition in politics? 10. What does the text insert entitled A Knee-Deep Crisis tell you about how the media covers the office of the presidency? Chapter 8 Multiple Choice 1. B 2. A 6. C 7. B 11. C 12. C 16. D 17. A 21. D 22. D 26. C 27. B True-False 1. T 6. F 2. F 7. F 3. B 8. C 13. D 18. C 23. B 28. C 3. T 8. F 4. D 9. A 14. B 19. D 24. D 29. A 4. F 9. F 5. B 10. B 15. D 20. B 25. A 30. B 5. T 10. T

164

CHAPTER NINE Who Wins, Who Loses: Pluralism Versus Elitism

CHAPTER OVERVIEW This chapter examines two competing schools of thought that attempt to define the essential structure and nature of the political game -- pluralism vs. an elite, ruling class--in order to answer several basic questions: Whos running the game of politics?, Who wins, who loses?, and Who plays and who doesnt play?. Pluralisms key concepts--fragmentation of power, bargaining, compromise, consensus--are covered, along with appropriate examples. Criticisms of pluralism include the question, What good are the rules of the game to the majority of people who never get a chance to play?, as well as the suggestion that an abundance of pluralism, i.e., too many groups, may explain present-day political paralysis. Similarly, the view that a powerful elite class dominates politics is explained, with stress placed upon this ruling class, especially C. Wright Mills model, of a unified group of leaders, drawn from the corporate, military, and political arenas. Critics of this view cite its over-reliance upon a conspiracy theory and point to a lack of unity within and among elites. The debate between the pluralist and elitist approaches centers upon how competitive and representative the elites are. The chapter closes with a discussion of recent modifications in the two theories, such as plural 165

elitism (politics divided into different policy arenas, with elite domination within each arena) and a call for greater participation in political life by the nations citizens. LEARNING OBJECTIVES After reading chapter 9, the student should be able to: 1. Explain and summarize the basic assumptions of pluralism, as well as the criticisms directed at the approach. 2. Explain and summarize the basic assumptions of the power elite model, as well as the criticisms directed at the approach. 3. Understand the application of these theoretical approaches and examples of both. 4. Explain modifications to pluralism and the power elite approaches, such as plural elitism. 5. Appreciate the importance of an active, politically concerned citizenry. OUTLINE I. Pluralism A. Pluralism Is a Group Theory of Democracy. People participate in politics by their membership in groups, and these groups, through competition and compromise, create public policy. There are four key concepts in the pluralist argument:

B.

166

1. Fragmentation of power. No one group is dominant; therefore, all must bargain. Power is divided, though not equally. 2. Bargaining. The government acts as a referee in this process. The government will make sure the rules of the game are followed and can intervene to help weaker groups. 3. Compromise. The inevitable result of a competition among relatively equal rivals is a series of compromises. Accommodation is made easier by the fact that most individuals are members of many groups. 4. Consensus. Underlying the entire process is a basic agreement on the general political ideals and goals of society. Agreement on rules and results is the cement that holds society together. Specific examples include agreeing on the importance of civil liberties and the goal of equality of opportunity. C. Examples of Pluralism. When major environmental groups decide that a new law regulating air pollution is needed, they raise funds from their members and seek compromises from chemical companies. The press weighs in and public opinion is heard, as in the EPA hearings before Congressional committees. The resulting legislation reflects the relative power of the groups. Similarly, Robert Dahls book, Who Governs?, concluded that there was no single elite that made important decisions in all areas of New Haven politics, but, rather, several different groups. Criticisms of Pluralist Theory. Many feel that consensus on democratic ideals masks the real inequity of economic and social distributions of benefits, that the majority of people have no part in the political game, and that powerful elites prevent issues from ever reaching the public. Other critics point to the 167

D.

political inflation of too many groups, choking government with too many demands (hyperpluralism). II. Elites This view asserts that the decision-making positions in America are occupied by members of a unified and non-representative elite class, the members of which look after their own interests. This power elite, of economic, political, and military leaders, presumably encourages powerlessness below and is unresponsive to others. A. How the Power Elite Rules. Power is centered in institutions. Therefore, key leadership positions in these institutions are reserved for the elite. These positions are open only to the ruling class of the nation. This class controls the economy and preserves the economic status quo. Decisions are the result not of consensus, but of conflict between the haves and have-nots and reflect the domination of the former. Society is held together by force and control, with elite domination of the majority a given. Examples of the Power Elite. In C. Wright Millss The Power Elite, the classic statement of the theory, the movement of leaders between the three dominant areas--business, the military, and government--is described. The Council on Foreign Relations is pointed to as an elite Establishment institution, controlling key foreign policy jobs and issues among its members. William Greiders Who Will Tell the People is a 1992 study of Washington politics, where a policymaking elite working with wealthy economic interests dominate the creating and implementing of law. In short, no one speaks for the people. Criticisms of the Elite View. Although it is true that the number of persons in positions of power is limited, their unity is exaggerated, and the elites do compete among themselves (examples are conflicts over taxes, the environment, the 168

B.

C.

election). In fact, democracy consists of people choosing among them, through the vote. Some say the elite are better suited to positions of leadership than are the masses. Often, elite views veer into conspiracy theories that do not analyze politics, but only assert secret cover-ups, which encourages apathy and cynicism among citizens, as well as fundraising by demagogues. III. The Debate/Newer Views Both approaches agree that only a small number of people participate directly in politics, and, therefore, the real question is how competitive and representative these elites are. The books case studies show how difficult it is to fit all real politics into one theory or the other. Recent modifications have discussed a plural elitism. This modification stresses that politics is divided into different policy arenas. Different political conflicts are understood by different approaches: for example, small-town politics may be best studied using a pluralist approach, and foreign policy through the elitist model. These models also reflect differing political ideals. Pluralism seeks to maintain the existing political structure, while power elite theorists maintain that basic changes are needed for the United States to become a true democracy. IV. Conclusion We do not have to be non-participants in the political game. People (including our students) can affect the outcome of politics, but they must, at the least, decide to join the game. Political conflict is not like a football game. KEY CHAPTER TERMS AND IDENTIFICATIONS Pluralism as a group theory of democracy fragmentation of power; bargaining, compromise, and consensus 169

"Government, under pluralism, reflects compromises among groups" Sierra Club/Clean Air Coalition Environmental Defense Fund; Environmental Protection Agency Robert Dahls Who Governs? (New Haven); and Democracy and Its Critics pluralism masks elite domination hyperpluralism power elite; power comes from institutions the ruling class; political decisions "represent conflict" The Council on Foreign Relations Foreign Affairs Old Boy Network C. Wright Mills The Power Elite William Greiders Who Will Tell the People the military-industrial complex a conspiracy theory elites--how competitive and representative? plural elitism (narrow elites dominate in separate policy areas) Pluralism and small communities; elite approach better for foreign policy Both elitists/pluralists ask/answer "what is" and "what should be" political conflict is not like a football game TEACHING SUGGESTIONS 1. Discuss with the class the following question: Are political elites more committed to democratic values than the mass public? Cite examples from recent events that support or refute this conclusion. 2. Another point to discuss: If the military-industrial complex does, in fact, exist, how can one fight it? Is it possible to organize effective opposition to new weapon systems and national defense spending? Have recent changes in defense policies, with the end of the Cold War, undermined the argument for the MIC? Also, was the plan by 170

President George W. Bush to reinvigorate the military, in 2001, an illustration of the MIC at workwhy or why not? 3. Discuss some conspiracy theories that are widely circulated. Do they play a political function for certain political leaders? Who circulates them? Why are the followers of these leaders, and others, so willing to accept them as credible? SUGGESTED RESEARCH ASSIGNMENTS 1. Have one student read Who Will Tell the People, by William Greider. Have him or her prepare a short, analytical book report and distribute copies of that report to the class. The student will then be responsible for defending the books approach to the rest of the class. 2. Follow the same procedure as described above, except that this time another student will report on Dahls Who Governs? It might be interesting to have the two students cross-examine each other, if classroom Q and A is insufficient. MULTIPLE-CHOICE QUESTIONS 1. In pluralism, people get power through a. voting. b. joining the military-industrial complex. c. becoming a member of various groups. d. joining a revolutionary party. Pluralism is a _____ theory of democracy. a. conspiratorial b. vague c. group d. participatory 171

2.

3.

In pluralist theory, the government acts as a(n) a. protector of the wealthy. b. referee. c. interest group. d. none of the above The following people are members of the Council on Foreign Relations: a. bankers b. lawyers c. academics d. all of the above The Council on Foreign Relations publishes which must read journal? a. The Economist b. Foreign Policy c. Pluralism Today d. Foreign Affairs The power elite theory of American government is a. a theory of the political left. b. a theory of the political right. c. held by both the left and right. d. held only by political scientists in colleges. Supporters of power elite theory view politics as a. a conflict between those with power and those without it. b. being a process based on coercion. c. a process under the domination of a ruling class. d. all of the above

4.

5.

6.

7.

172

8.

Consensus is best described as a. the result of political compromises. b. common sense. c. a broad public agreement on basic political questions. d. the people of a country voting frequently. Power elite theorists see the ruling elite as a. unrepresentative. b. unified. c. encouraging powerlessness in non-elites. d. all of the above

9.

10. According to Robert Dahl, in Who Governs?, which is true? a. a power elite is a crucial engine for social change. b. pluralism conceals elite manipulation. c. pluralism allows a charismatic leader to corrupt democracy. d. different groups influence decisions in different policy arenas. 11. In the pluralist model, bargaining results in a series of compromises that become _____ a. another series of compromises. b. the basis of a new political consensus. c. public policy or other political decisions. d. none of the above 12. According to Robert Dahls study, the best description of key decision-makers in New Haven was that of a. a unified, homogeneous elite. b. basically, the citizenry of the city. c. members of the city council. d. none of the above

173

13. In the United States the political stratum does not constitute a homogeneous class with well-defined class interests. This assertion is associated with the _____ approach. a. pluralist b. power elite c. plural elitism d. hyperpluralism 14. In the power elite approach, power comes mainly from a. individuals. b. institutions. c. the mass public. d. none of the above 15. Which of the following does not describe hyperpluralism? a. a stable political system b. a critique of pluralism c. too many competing groups d. a government paralyzed by too many demands 16. Which of the following is a criticism of pluralist theory? a. It confuses equal opportunity with equality. b. It ignores non-participants. c. It stresses procedures rather than results. d. All of the above are criticisms. 17. The AIDS epidemic that broke out in the 1980s is cited by a. conspiracy theorists, as an example of a secret act of revenge. b. critics of pluralism, who feel it was a failure of democracy. c. critics of power elite theory, to demonstrate how some elites check others. d. all of the above

174

18. In the debate between the two approaches, the central disagreement concerns a. whether elites exist. b. whether the elites are representative and competitive. c. whether people directly participate in the political game. d. none of the above 19. In both pluralism and elitism, government actions are produced by a. forces generally external to the government. b. the result of irrational, corrupt forces. c. the consensus of society. d. the manipulation of a unified elite. 20. Politics is compared to the original primitive game of football because a. there are few rules of behavior and conduct attached to politics, so participants can do anything to achieve political results. b. the football audience (the public) is free to join in the game. c. increasingly, the game of politics is scarred by violence directed at elected officials. d. all of the above are true 21. In the chapter example, environmentalists proposed changes in air pollution laws resulted in a. a veto by the hidden power elite in America. b. debate, challenge, and cooperation by a number of concerned groups. c. minimum debate and dismissal by Congress. d. none of the above

175

22. Under pluralism, a member of two interest groups competing over an issue--like health care or the environment--would likely a. resign his or her membership from both groups. b. decide to remain neutral on the issue. c. try to have both groups reach a compromise. d. none of the above 23. In the previous case study of the decline and fall of racial segregation, its resolution seemed to be most accurately reflected in a. the approach of pluralism. b. the approach of the power elite. c. the new approach of plural elitism. d. the approach of hyperpluralism. 24. The author feels that a greater degree of political participation by Americans would mean that a. traditional family values could be reinstated. b. changes could occur in how the political game is played and its outcome. c. little has changed in the political game. d. none of the above 25. Plural elitism means that a. politics is divided into different policy arenas. b. special interest elites dominate in specific arenas. c. the general public doesnt clearly see its interest. d. all of the above TRUE-FALSE QUESTIONS 1. 2. Bargaining and group membership are essential elements of pluralist theory. Fragmentation of power ensures that power is divided equally. 176

3. 4. 5. 6. 7. 8. 9.

According to critics of pluralism, powerful elites may prevent important issues from reaching the public arena for a decision. Most Americans are participants in the game of politics. Plural elitism suggests that the majority of people can effectively participate in politics. Conspiracy theorists generally reject the view that government is a reflection of group compromises. Power elite supporters argue that members of the Council on Foreign Relations do not represent the varied interests in American society. Pluralists and elitists are both asking not only what is, but what should be. The fundamental issue in the pluralist-elite debate is whether a small number of people dominate the political game.

10. The texts earlier case studies clearly reveal that the political game can be placed under a "single umbrella of ideas." ESSAY QUESTIONS 1. How would pluralists argue that a presidential election reflects their approach? Give examples from the recent campaign and election. 2. How would the power elite approach argue that this same election reinforces their view of American politics? Give specific examples.

177

3. In your town, city, or community, which approach, or mixture of approaches, best explains the political decisions with which you are familiar? Support your answer with details and examples. 4. Now that you have finished reading the text, speculate on how more Americans can become participants in the political system. Among your own fellow students, what techniques could be used to promote interest about political issues and personalities? 5. Take any two case studies from the text and apply the pluralist approach to one, and the power elite approach to the other. Which makes the most sense of the material covered? Chapter 9 Multiple Choice 1. C 2. C 6. C 7. D 11. C 12. D 16. D 17. A 21. B 22. C True-False 1. T 6. T 2. F 7. T 3. B 8. C 13. A 18. B 23. A 3. T 8. T 4. D 9. D 14. B 19. A 24. B 4. F 9. F 5. D 10. D 15. A 20. B 25. D 5. F 10. F

178

You might also like